exam 1 chapter questions ob

Lakukan tugas rumah & ujian kamu dengan baik sekarang menggunakan Quizwiz!

A pregnant patient would like to know a good food source of calcium other than dairy products. Your best answer is: A) Legumes B) Yellow vegetables C) Lean meat D) Whole grains

A) Legumes

When providing care to the prenatal patient, the nurse understands that pica is defined as: A) Intolerance of milk products B) Iron deficiency anemia C) Ingestion of nonfood substances D) Episodes of anorexia and vomiting

C) Ingestion of nonfood substances

A woman has a breast mass that is not well delineated and is nonpalpable, immobile, and nontender. This is most likely: A) Fibroadenoma. B) Lipoma. C) Intraductal papilloma. D) Mammary duct ectasia.

C) Intraductal papilloma.

The prenatal medical record for a pregnant patient with diabetes states that amniotic fluid phosphatidylglycerol is greater than 3%. What does this indicate? 1 The fetal lung maturation is normal. 2 The mother may develop hydramnios. 3 There is a chance of fetal macrosomia. 4 The mother is at risk for hypoglycemia

1 Fetal lung maturation is predicted by the presence by amniotic fluid phosphatidylglycerol. If it is greater than 3%, the fetal lung maturation is normal. The risk for hypoglycemia can be confirmed by evaluating blood glucose levels. The risk for fetal macrosomia is increased if there is poor glycemic control in pregnancy. If hyperglycemia occurs, the patient is at risk for hydramnios. Test-Taking Tip: Survey the test before you start answering the questions. Plan how to complete the examination in the time allowed. Read the directions carefully and answer the questions you know for sure first.

Which action does the nurse take to determine whether the carbohydrate intake is inadequate in a pregnant patient with diabetes? 1 Monitor for urine ketones. 2 Evaluate the nonstress test results. 3 Determine the degree of glycosuria. 4 Schedule a baseline fetal sonogram.

1 If a patient with diabetes does not take in enough carbohydrates, the body resorts to breaking down fats for energy. The by-product of fat metabolism is ketones. Therefore the nurse monitors the urine for ketones. The amount of ketones in the urine helps detect inadequate carbohydrate intake. Nonstress tests will help assess the well-being of the fetus. Glycosuria does not accurately reflect the blood glucose levels because of a lowered renal threshold for glucose during pregnancy. The nurse obtains a baseline sonogram to assess gestational age in the first trimester. Test-Taking Tip: Relax during the last hour before an examination. Your brain needs some recovery time to function effectively.

Which condition does the nurse expect to find in the medical record of a pregnant patient with diabetes mellitus? 1 Hypoglycemia 2 Cyanosis of lips 3 Increasing fatigue 4 Generalized edema

1 In the first trimester of pregnancy, there is an increase in insulin production and a decrease in hepatic glucose production because of the rising levels of estrogen and progesterone. This lowers the glucose levels and makes the pregnant patient prone to hypoglycemia. Cyanosis of lips, increasing fatigue, and generalized edema are signs of cardiac decompensation in a pregnant patient with preexisting cardiac disease. It occurs because the heart is unable to maintain sufficient cardiac output because of pregnancy.

Which factor is known to increase the risk of gestational diabetes mellitus? 1 Previous birth of large infant 2 Maternal age younger than 25 3 Underweight before pregnancy 4 Previous diagnosis of type 2 diabetes mellitus

1 Previous birth of a large infant suggests gestational diabetes mellitus. A woman younger than 25 is not at risk for gestational diabetes mellitus. Obesity (greater than 90 kg or 198 lb) creates a higher risk for gestational diabetes. The person with type 2 diabetes mellitus already has diabetes and will continue to have it after pregnancy. Insulin may be required during pregnancy because oral hypoglycemia drugs are contraindicated during pregnancy. Test-Taking Tip: Be alert for details. Details provided in the stem of the item, such as behavioral changes or clinical changes (or both) within a certain time period, can provide a clue to the most appropriate response or (in some cases) responses.

Which minerals and vitamins usually are recommended to supplement a pregnant womans diet? A) Fat-soluble vitamins A and D B) Water-soluble vitamins C and B6 C) Iron and Folate D) Calcium and zinc

C) Iron and Folate

With regard to protein in the diet of pregnant women, nurses should be aware that: A) Many protein-rich foods are also good sources of calcium, iron, and B vitamins. B) Many women need to increase their protein intake during pregnancy. C) As with carbohydrates and fat, no specific recommendations exist for the amount of protein in the diet. D) High-protein supplements can be used without risk by women on macrobiotic diets.

A) Many protein-rich foods are also good sources of calcium, iron, and B vitamins.

What does the nurse include in the plan of care of a patient with a cardiac disorder during the postpartum period? Select all that apply. 1 Monitor oxygen saturation levels. 2 Place the newborn at the bedside. 3 Put the patient on a full liquid diet. 4 Teach the patient how to breastfeed. 5 Have the patient talk to the newborn.

1, 2, 4, 5 The nurse monitors oxygen saturation levels in the patient to assess for adequate oxygenation. The nurse places the infant at the bedside so that the patient can touch the infant without expending energy. This also helps establish an emotional bond. The nurse assists the patient in breastfeeding by positioning the infant correctly for feeding. The nurse encourages the patient to talk to the newborn to involve the mother in the infant's care and help the patient feel vitally important. A fluid diet is not prescribed, because it does not ensure adequate nutrition for the infant.

The nurse is developing the plan of care for a pregnant patient with an underlying history of cardiovascular disease. Which conditions would this patient be at risk for? Select all that apply. 1 Stillbirth 2 Miscarriage 3 Hypoglycemia 4 Atrial septal defect 5 Intrauterine growth restriction

1, 2, 5 Stillbirth and miscarriage may occur because of cardiovascular problems in pregnancy. Intrauterine growth restriction results in a pregnant patient with cardiovascular disease because of low oxygen pressure. Hypoglycemia is seen in a pregnant patient with diabetes because a decrease in glucose levels. An atrial septal defect is a congenital birth defect that is not related to underlying maternal cardiac disease.

A pregnant woman is being examined by the nurse in the outpatient obstetric clinic. The nurse suspects systemic lupus erythematosus (SLE) after the examination reveals which symptoms? Select all that apply. 1 Muscle aches 2 Hyperactivity 3 Weight changes 4 Fever 5 Hypotensio

1, 3, 4 Common symptoms, including myalgias, fatigue, weight change, and fevers, occur in nearly all women with SLE at some time during the course of the disease. Fatigue, rather than hyperactivity, is a common sign of SLE. Hypotension is not a characteristic sign of SLE. Although a diagnosis of SLE is suspected based on clinical signs and symptoms, it is confirmed by laboratory testing that demonstrates the presence of circulating autoantibodies. As with other autoimmune diseases, SLE is characterized by a series of exacerbations (flares) and remissions (Chin and Branch, 2012).

Which interventions does the nurse implement to ease the labor process of a pregnant patient with heart disease? Select all that apply. 1 Place the patient in a side-lying position. 2 Use stirrups to facilitate an easy labor. 3 Monitor the patient's oxygen saturation. 4 Maintain a peaceful, calm environment. 5 Provide the patient with a relaxing back rub.

1, 3, 4, 5 The nurse places the patient in a side-lying position to facilitate uterine perfusion. The nurse monitors the patient's oxygen saturation to assess for adequate oxygenation. The nurse maintains a calm environment to minimize the patient's anxiety. The nurse also provides a back massage to comfort the patient. Stirrups are not used because they may prevent compression of the popliteal veins or increase in blood volume in the chest.

Which is the ideal treatment for severe unmanageable hyperthyroidism in a patient who is pregnant? 1 Radioactive iodine 2 Subtotal thyroidectomy 3 Methimazole (Tapazole) 4 Propylthiouracil (Propacil

2 A subtotal thyroidectomy is prescribed for a pregnant patient with severe hyperthyroidism if the drug therapy proves toxic. Oral methimazole and propylthiouracils are prescribed for hyperthyroidism but may be ineffective in severe cases. Radioactive iodine is not used to treat hyperthyroidism in pregnant patients, because it may destroy the fetus's thyroid gland.

Diabetes in pregnancy puts the fetus at risk in several ways. Nurses should be aware that: 1 with good control of maternal glucose levels, sudden and unexplained stillbirth is no longer a major concern. 2 the most important cause of perinatal loss in diabetic pregnancy is congenital malformations. 3 infants of mothers with diabetes have the same risks for respiratory distress syndrome because of the careful monitoring. 4 at birth, the neonate of a diabetic mother is no longer at any greater risk.

2 Congenital malformations account for 30% to 50% of perinatal deaths. Even with good control, sudden and unexplained stillbirth remains a major concern. Infants of diabetic mothers are at increased risk for respiratory distress syndrome. The transition to extrauterine life often is marked by hypoglycemia and other metabolic abnormalities.

When a pregnant woman with diabetes experiences hypoglycemia while hospitalized, what should the nurse have the woman do? 1 Eat a candy bar. 2 Eat six saltine crackers or drink 8 oz of milk. 3 Drink 4 oz of orange juice followed by 8 oz of milk. 4 Drink 8 oz of orange juice with 2 teaspoons of sugar added

2 Crackers provide carbohydrates in the form of polysaccharides. A candy bar provides only monosaccharides. Milk is a disaccharide and orange juice is a monosaccharide. This will provide an increase in blood sugar but will not sustain to level. Orange juice and sugar will increase the blood sugar, but not provide a slow-burning carbohydrate to sustain the blood sugar. Test-Taking Tip: Being prepared reduces your stress or tension level and helps you maintain a positive attitude.

A pregnant woman with cardiac disease is informed about signs of cardiac decompensation. She should be told that the earliest sign of decompensation is most often: 1 orthopnea. 2 decreasing energy levels. 3 moist frequent cough and frothy sputum. 4 crackles (rales) at the bases of the lungs on auscultation.

2 Decreasing energy level (fatigue) is an early finding of heart failure. Care must be taken to recognize it as a warning rather than a typical change of the third trimester. Cardiac decompensation is most likely to occur early in the third trimester, during childbirth, and during the first 48 hours following birth. Orthopnea, a moist, frequent cough, and crackles and rales appear later when a failing heart reduces renal perfusion and fluid accumulates in the pulmonary interstitial space, leading to pulmonary edema.

How does the nurse advise the patient who has given birth to an infant with microcephaly in the past and is now planning for the next child? 1 "There is a higher chance of having a preterm birth." 2 "You should be screened for phenylketonuria (PKU)." 3 "There may be a miscarriage in your second pregnancy." 4 "You must go for genetic counseling before conception."

2 If a patient has given birth to an infant with microcephaly in the past, there is a possibility that the patient has phenylketonuria (PKU). PKU results from a deficiency in the enzyme phenylalanine hydrolase. Preterm birth is a possibility in pregnant women with untreated hypothyroidism. PKU affects brain development and function in the child; it does not cause miscarriage. Genetic counseling is more important for patients who have hereditary disorders, which can be passed on to the child. Test-Taking Tip: Because few things in life are absolute without exceptions, avoid selecting answers that include words such as always, never, all, every, and none. Answers containing these key words are rarely correct.

Which medication is administered to a pregnant patient to treat hyperthyroidism? 1 Isotretinoin (Accutane) 2 Methimazole (Tapazole) 3 Levothyroxine (Synthroid) 4 Sodium iodide 131I (Hicon

2 Methimazole (Tapazole) is administered to a pregnant patient to control symptoms of hyperthyroidism, improve weight gain, and reduce tachycardia. Isotretinoin (Accutane) is prescribed for cystic acne. It is not prescribed during pregnancy, because it is highly teratogenic. Levothyroxine (Synthroid) is used to treat hypothyroidism in pregnant patients who do not have a functioning thyroid tissue. Sodium iodide 131I (Hicon) is radioactive iodine. It is not used to treat hyperthyroidism in pregnant patients, because it may destroy the fetal thyroid. Test-Taking Tip: Eat breakfast or lunch before an examination. Avoid greasy, heavy foods and overeating. This will help keep you calm and give you energy.

Which clinical finding in a pregnant patient will indicate proper fetal brain development? 1 Hemoglobin A1c levels greater than 6 2 Normal maternal thyroxine (T4) levels 3 3% amniotic fluid phosphatidylglycerol 4 Fasting glucose levels less than 95 mg/dL

2 Proper fetal brain development depends on normal maternal T4 levels early in pregnancy. Mild maternal hypothyroidism during the first trimester can cause neuropsychological damage in the infant. Hemoglobin A1c levels greater than 6 indicate long-term elevated glucose levels in the patient. A 3% amniotic fluid phosphatidylglycerol indicates proper lung maturation in the fetus. Fasting glucose levels less than 95 mg/dL indicate proper glycemic control in the pregnant patient.

From 4% to 8% of pregnant women have asthma, making it one of the most common preexisting conditions of pregnancy. Severity of symptoms usually peaks: 1 in the first trimester. 2 between 17 to 24 weeks of gestation. 3 during the last 4 weeks of pregnancy. 4 immediately postpartum

2 The period between 17 and 24 weeks of pregnancy is associated with the greatest severity of symptoms. Women often have few symptoms of asthma during the first trimester. During the last 4 weeks of pregnancy symptoms often subside. Often issues have resolved by the time the woman gives birth.

A patient has been prescribed adjuvant tamoxifen therapy. What common side effect might she experience? A) Nausea, hot flashes, and vaginal bleeding B) Vomiting, weight loss, and hair loss C) Nausea, vomiting, and diarrhea D) Hot flashes, weight gain, and headaches

A) Nausea, hot flashes, and vaginal bleeding

A pregnant woman with type 1 diabetes is on rapid-acting, short-acting, and intermediate-acting insulin injections. Which are rapid and short-acting insulins? Select all that apply. 1 NPH (Novolin N) 2 Regular (Humalin) 3 Lispro (Humalog) 4 Aspart (NovoLog) 5 Glargine (Lantus)

2, 3, 4 Humalog and NovoLog are rapid-acting insulins and Humalin is a short-acting insulin. Novolin N is an intermediate-acting insulin and Lantus is a long-acting insulin.

A patient with gestational diabetes tells the nurse, "I'm extremely worried that my child will be diabetic, too." Which actions does the nurse take to alleviate the patient's anxiety? Select all that apply. 1 Evaluate the test results to assess fetal growth. 2 Listen to the feelings and concerns of the patient. 3 Provide factual information of risks to the patient. 4 Use therapeutic communication with the patient. 5 Ask the patient to share any fears with the nurse.

2, 3, 4, 5 The nurse listens to the patient's feelings and concerns to assess for any misconception or misinformation that can be causing anxiety. The nurse provides factual information about any risks to the patient to correct any misconceptions. Using therapeutic communication will develop an open relationship that also helps promote trust. The nurse encourages the patient to share concerns with the nursing staff to promote collaboration in the care process. Evaluating test reports for fetal growth will help assess fetal well-being, although it does not help alleviate the patient's fears.

What does the nurse recommend to a pregnant patient with diabetes who works long, irregular hours? 1 "Eat a snack hourly when at work." 2 "Try taking naps when you are free." 3 "Keep fruits or fruit juice available." 4 "Quit working for a while."

3 If the patient has to be away from home for long hours, the nurse advises the patient to carry fruits or fruit juices. They contain simple carbohydrates that help control blood glucose levels. The nurse should not advise the patient to quit working, because it may not be feasible for the patient. Instead, the nurse should encourage the patient to follow a consistent daily schedule. Taking naps when free ensures rest, but it does not help keep glucose levels in check. Eating a snack every hour is not advised, because it may fluctuate blood glucose levels. Instead, three meals and two or three snacks are advised.

During pregnancy, alcohol withdrawal may be treated using: 1 disulfiram (Antabuse). 2 corticosteroids. 3 benzodiazepines. 4 aminophylline.

3 Symptoms that occur during alcohol withdrawal can be managed with short-acting barbiturates or benzodiazepines. Disulfiram is contraindicated in pregnancy because it is teratogenic. Corticosteroids are not used to treat alcohol withdrawal. Aminophylline is not used to treat alcohol withdrawal. Test-Taking Tip: Identify option components as correct or incorrect. This may help you identify a wrong answer. Example: If you are being asked to identify a diet that is specific to a certain condition, your knowledge about that condition would help you choose the correct response (e.g., cholecystectomy = low-fat, high-protein, low-calorie diet).

Which condition should the nurse be alert for after administering terbutaline (Brethine) to a pregnant patient with diabetes mellitus? 1 Dyspnea 2 Infection 3 Ketoacidosis 4 Hypoglycemia

3 Terbutaline (Brethine) is a beta-mimetic drug administered for tocolysis to stop preterm labor. It may lead to hyperglycemia and cause ketoacidosis in the pregnant patient. Dyspnea on exertion may be seen in a pregnant patient with acquired cardiac disease. Infection in pregnant women happens because of an alteration in the normal resistance of the body to infection. Hypoglycemia occurs if there is an increase in the insulin levels.

What instruction should the nurse give to a pregnant patient with hyperthyroidism who often gets fatigued and weak as a result of nervousness and hyperactivity? 1 "Ensure that you wear warm clothes." 2 "Perform aerobic exercises every day." 3 "Become involved in reading or a craft." 4 "Avoid going out in the cold or at night."

3 The nurse advises the patient to engage in quiet activities, such as reading or crafting, to prevent fatigue and weakness. Extreme cold temperature is prevented and warm clothing is suggested if the patient has cold intolerance because of hypothyroidism. The patient is hypersensitive to heat and gets easily fatigued; therefore aerobic exercises are not advised.

A patient is diagnosed with severe cardiac disease in the fourth week of pregnancy. What should be the nurse's priority? 1 Initiate fetal surveillance twice a week. 2 Assess the patient for signs of heart failure. 3 Explain the risks of continuing pregnancy. 4 Tell the patient to end the pregnancy right away.

3 The nurse explains the risks of continuing the pregnancy to help the patient make an informed decision. Fetal surveillance begins after 28 weeks if there are any complications, such as diabetes mellitus, in the patient. The nurse informs the patient about the complications that may arise as the pregnancy progresses. If the pregnancy is continued, the nurse assesses for cardiac decompensation to evaluate if the heart is able to maintain a sufficient cardiac output. The nurse does not ask the patient to terminate the pregnancy.

The nurse is teaching a woman with gestational diabetes the technique to inject insulin. What should the nurse include in the teaching session? Select all that apply. 1 Aspirate before injecting. 2 Clean injection site with alcohol. 3 Insert the needle at a 45- to 90-degree angle. 4 Inject insulin slowly. 5 After injection, cover site with sterile gauze.

3, 4, 5 Insulin should be injected with the short needle inserted at a 45- to 90-degree angle, depending on fatty tissue. Insulin is injected slowly to allow tissue expansion and minimize pressure, which can cause insulin leakage. After injection, the site should be covered with sterile gauze. Gentle pressure should be applied to prevent bleeding. Aspirating when injecting into subcutaneous tissue is not necessary. The injection site should be clean, but using alcohol is not necessary.

Which medication does the primary health care provider ask the nurse to administer to a patient during labor in a vaginal delivery, who has a history of a myocardial infarction (MI)? 1 Oxytocin 2 Diuretics 3 Anticoagulant 4 Epidural analgesia

4 Epidural analgesia is administered during labor to a patient with MI to prevent pain, which can result in tachycardia and increased cardiac demands. Oxytocin is administered to a patient after birth to prevent hemorrhage. Diuretics are administered to prevent fluid retention in a pregnant patient with a heart disease. Anticoagulant therapy is administered for recurrent venous thrombosis in pregnancy. Test-Taking Tip: Notice how the subjects of the questions are related and, through that relationship, the answers to some of the questions you skipped may be provided within other questions of the test.

Which medication is ideal for the treatment of systemic lupus erythematosus (SLE) in a pregnant patient? 1 Aspirin (Ecotrin) 2 Azathioprine (Imuran) 3 Prednisone (Deltasone) 4 Hydroxychloroquine (Plaquenil)

4 Hydroxychloroquine (Plaquenil) reduces SLE disease activity in a pregnant patient without any adverse effects on the fetus. Aspirin (Ecotrin) is not recommended during pregnancy, because it has an increased risk for premature closure of the fetal ductus arteriosus. Azathioprine (Imuran) is discontinued before conception, because it is fetotoxic. Prednisone (Deltasone) is prescribed to treat SLE during pregnancy, but it increases the risk for bone demineralization, gestational diabetes, preeclampsia, premature rupture of membranes (PROM), and intrauterine growth restriction (IUGR).

A pregnant woman at 28 weeks of gestation has been diagnosed with gestational diabetes. The nurse caring for this woman understands that: 1 oral hypoglycemic agents should not be used if the woman is reluctant to give herself insulin. 2 dietary modifications and insulin are both required for adequate treatment. 3 glucose levels are monitored by testing urine 4 times a day and at bedtime. 4 dietary management involves distributing nutrient requirements over three meals and two or three snacks.

4 Small frequent meals over a 24-hour period help decrease the risk for hypoglycemia and ketoacidosis. Oral hypoglycemic agents can be used as an alternative to insulin in women with GDM who require medication in addition to diet for blood glucose control. In some women gestational diabetes can be controlled with dietary modifications alone. Blood, not urine, glucose levels are monitored several times a day. Urine is tested for ketone content; results should be negative.

What does the nurse inform a breastfeeding patient who is taking propylthiouracil (Propacil) for hyperthyroidism? 1 "The medication is likely to decrease milk production." 2 "Stop breastfeeding the child, and start infant formula." 3 "It can adversely affect the neonate's thyroid function." 4 "Take the medication immediately after breastfeeding."

4 The nurse advises the patient to take the medication immediately after breastfeeding to allow a 3- to 4-hour period for the medication to absorb before nursing again. Milk production decreases if there is poor metabolic control, not because of antithyroid medications. It is not necessary to stop breastfeeding or provide infant formula, because there are no side effects of the medication in the infant. The medication also does not adversely affect the neonate's thyroid function because the infant has a normal thyroid function.

Most women with uncomplicated pregnancies can use the nurse as their primary source for nutritional information. The nurse or midwife should refer a client to a registered dietitian for in-depth nutritional counseling in the following situations (Select all that apply). A) Preexisting or gestational illness such as diabetes B) Ethnic or cultural food patterns C) Obesity D) Vegetarian diet E) Allergy to tree nuts

A) Preexisting or gestational illness such as diabetes B) Ethnic or cultural food patterns C) Obesity D) Vegetarian diet

4 Karyotyping is the estimation of the number, size, and forms of chromosomes for detecting chromosomal abnormalities like trisomy. These are the major causes for frequent miscarriage, stillbirths, and other gynecologic disorders. Blood sample is the preferred sample for determining chromosome abnormalities. Nucleated and replicating cells such as white blood cells are extracted from the blood sample and chosen for karyotyping. Urine, saliva, and muscle samples are not used for analyzing chromosome number, because they do not undergo rapid division to form new cells.

A patient who has experienced frequent miscarriages is referred for chromosomal analysis. Which sample should the nurse obtain to conduct karyotyping for the patient? 1 Urine 2 Saliva 3 Muscle 4 Blood

3 Cystic fibrosis is an autosomal recessive disorder. It is inherited and expressed only when the individual carries both the genes for the abnormality. Because both parents are carriers, both of them contributed their recessive genes to the child. It is an autosomal recessive disorder and is not associated with X-linked recessive or dominant inheritance. Autosomal dominant disorders need only one copy of the gene for variation. If the gene for variation is carried on the dominant allele, the individual exhibits the disorder.

A child born to a couple is diagnosed with cystic fibrosis. After reviewing the genetic reports, the couple is found to be carriers for the disorder. How should the nurse explain this to the couple? This disorder is an: 1 "X-linked recessive inherited disorder." 2 "X-linked dominant inherited disorder." 3 "Autosomal recessive inherited disorder." 4 "Autosomal dominant inherited disorder."

3 Karyotyping provides genetic information, such as gender and chromosomal structure. The lecithin/sphingomyelin ratio, not karyotyping, reveals lung maturity. Although karyotyping can detect genetic anomalies, the range of normal is nondescriptive. Although karyotyping can detect genetic anomalies, not all such anomalies display obvious physical deformities. The term deformities is a nondescriptive word. Furthermore, physical anomalies may be present that are not detected by genetic studies (e.g., cardiac malformations).

A couple has been counseled for genetic anomalies. They ask, "What is karyotyping?" The nurse's best response is: 1 "Karyotyping will reveal if the baby's lungs are mature." 2 "Karyotyping will reveal if your baby will develop normally." 3 "Karyotyping will provide information about the gender of the baby, and the number and structure of the chromosomes." 4 "Karyotyping will detect any physical deformities the baby has.

2 Hemophilia is an X-linked recessive inheritance disorder carried on the X chromosome. In the scenario, the patient is homozygous to hemophilia because the symptoms are expressed in the patient. The patient's partner is not a carrier for hemophilia. The abnormal trait present on the X chromosome of the mother will be expressed in the male offspring, because males receive X chromosome from the mother. Therefore the chance of having hemophilia in the male offspring is 100%. If the mother is a carrier for hemophilia, then there is a 50% chance of hemophilia in the male offspring. All the female offspring of the patient will be carriers for hemophilia, because the partner does not carry any abnormal X chromosome for hemophilia.

A genetic test in a patient with hemophilia shows that the patient's partner is not a carrier for hemophilia. The patient is worried and wants to know about the chances of hemophilia in their children. What information should the nurse convey to the patient? The chance of hemophilia in the: 1 "Male offspring is 50%." 2 "Male offspring is 100%." 3 "Female offspring is 50%." 4 "Female offspring is 100%."

3, 4, 5 The nurse explains to the couple and their family members about the cause of the disorder. The disorder is caused by a nondisjunction of sex chromosomes and the child is not responsible for it. Genetic counseling for the family may provide detailed knowledge of the syndrome and the possibility of future children to have the syndrome. This may help the family understand the disorder more effectively. Understanding the symptoms and treatment methods would help the parents and the family members adjust according to the needs of the child. The nurse needs to motivate the family to accept the child and not give the child up for adoption or find places to leave the child. The nurse also explains the child's need for emotional support during growing years and that it is the right of the child to be with parents.

A newborn is diagnosed with Klinefelter syndrome. On assessing the family, the nurse finds that the couple feels embarrassed by the child. How should the nurse help the couple in coping with the situation? Select all that apply. The nurse: 1 Suggests giving up the child for adoption. 2 Explains the main reason for the disorder. 3 Directs the family to a genetic counselor. 4 Educates the family about the disorder. 5 Instructs about places to leave the child

1 Scalp hair appears after week 16 of pregnancy, along with eyes, ears, and the nose. However, it is still in the developmental stages and is observed upon gross examination of the fetus. Sweat glands are formed at the twenty-fourth week of pregnancy. Nasal cartilage is observed after 40 weeks of pregnancy. Sebaceous glands are formed after 20 weeks of pregnancy.

A patient in the eighteenth week of pregnancy visits a clinic to know about the development of the fetus. The patient says to the nurse, "I am excited, and I want to know what has developed in my child so far." What information should the nurse be able to tell to the patient about the fetus? The fetus has: 1 "Hair on the scalp." 2 "Some sweat glands." 3 "Nasal cartilage." 4 "Sebaceous glands."

Which symptom described by a patient is characteristic of premenstrual syndrome (PMS)? A) I feel irritable and moody a week before my period is supposed to start. B) I have lower abdominal pain beginning the third day of my menstrual period. C) I have nausea and headaches after my period starts, and they last 2 to 3 days. D) I have abdominal bloating and breast pain after a couple days of my period.

A) I feel irritable and moody a week before my period is supposed to start.

1 Fetal movements are generally observed between 16 to 20 weeks of gestation. As the patient is in the 8th week of gestation, she will be able to notice fetal movements in another 8 to 12 weeks. By this time the arms and legs of the fetus will be completely developed. All the other options, which are below 16 weeks of gestation are incorrect as the arms and legs of the fetus are not well developed. Therefore, the patient cannot feel the fetal movements after 6 to 8 weeks, 4 to 6 weeks, or 2 to 4 weeks from then.

A patient who is 8 weeks pregnant wants to know when she can feel the fetal movements in the womb. What is the best nursing response? "You will have to wait for another: 1 8 to 12 weeks." 2 6 to 8 weeks." 3 4 to 6 weeks." 4 2 to 4 weeks."

1 A parent may show no symptoms of an autosomal recessive condition but still carry a gene mutation for that condition. A carrier screening test can be performed to rule out the possibility of the fetus being affected by the condition. Maternal serum screening is a blood test that is helpful in detecting the presence of neural tube defects and chromosomal abnormalities such as Down syndrome in the fetus; it is not used to determine if the parent is a carrier of an autosomal recessive condition. Predispositional predictive testing is performed to test for mutations, such as those involving the BRCA1 gene to determine susceptibility for breast cancer; it is not used to determine if the parent is a carrier of an autosomal recessive condition. Presymptomatic predictive testing is done for the mutation analysis for Huntington disease (HD), not to determine if the parent is a carrier of an autosomal recessive condition.

A pregnant patient has a sister with a genetic disorder but has never shown symptoms of the disorder herself. The patient's husband has an uncle with the disorder but has likewise never shown symptoms of the disorder himself. Which test is performed to determine the risk of the patient's fetus acquiring this disorder? 1 Carrier screening test 2 Maternal serum screening 3 Predispositional predictive testing 4 Presymptomatic predictive testing

3 The detection of the presence of pulmonary surfactants (surface-active phospholipids) in amniotic fluid has been used to determine fetal lung maturity, or the ability of the lungs to function after birth. This occurs at approximately 35 weeks of gestation. The presence of surface-active phospholipids is not an indication of Down syndrome. This result reveals the fetal lungs are mature and in no way indicates risk for preterm labor. Meconium should not be present in the amniotic fluid.

A woman at 35 weeks of gestation has had an amniocentesis. The results reveal that surface-active phospholipids are present in the amniotic fluid. The nurse is aware that this finding indicates: 1 the fetus is at risk for Down syndrome. 2 the woman is at high risk for developing preterm labor. 3 lung maturity. 4 meconium is present in the amniotic fluid.

2 Fetuses respond to sound by 24 weeks. The fetus can be soothed by the sound of the mother's voice. Although stating that many pregnant women imagine what their baby is like is accurate, it is not the most appropriate response. The mother should be instructed that her fetus can hear at 24 weeks and can respond to the sound of her voice. Stating that if the woman thinks that her baby hears will help them bond gives the impression that her baby cannot hear her. It also belittles the mother's interpretation of her fetus's behaviors.

A woman is 8 months pregnant. She tells the nurse that she knows her baby listens to her, but her husband thinks she is imagining things. Which response by the nurse is most appropriate? 1 "Many pregnant women imagine what their baby is like." 2 "A baby in utero does respond to the mother's voice." 3 "You'll need to ask the doctor if the baby can hear yet." 4 "Thinking that your baby hears will help you bond with the baby."

Which statement about pregnancy is accurate? A) A normal pregnancy lasts about 10 lunar months. B) A trimester is one third of a year. C) The prenatal period extends from fertilization to conception. D) The estimated date of confinement (EDC) is how long the mother will have to be bedridden after birth.

A) A normal pregnancy lasts about 10 lunar months.

Which nutritional recommendation about fluids is accurate? A) A woman's daily intake should be eight to ten glasses (2.3 L) of water, milk, or juice. B) Coffee should be limited to no more than two cups, but tea and cocoa can be consumed without worry. C) Of the artificial sweeteners, only aspartame has not been associated with any maternity health concerns. D) Water with fluoride is especially encouraged because it reduces the child's risk of tooth decay.

A) A woman's daily intake should be eight to ten glasses (2.3 L) of water, milk, or juice.

Prenatal testing for human immunodeficiency virus (HIV) is recommended for: A) All women, regardless of risk factors. B) A woman who has had more than one sexual partner. C) A woman who has had a sexually transmitted infection. D) A woman who is monogamous with her partner.

A) All women, regardless of risk factors.

On vaginal examination of a 30-year-old woman, the nurse documents the following findings: profuse, thin, grayish white vaginal discharge with a fishy odor; complaint of pruritus. On the basis of these findings, the nurse suspects that this woman has: A) Bacterial vaginosis (BV) B) Candidiasis C) Trichomoniasis D) Gonorrhea

A) Bacterial vaginosis (BV)

With regard to medications, herbs, shots, and other substances normally encountered by pregnant women, the maternity nurse should be aware that: A) Both prescription and over-the-counter (OTC) drugs that otherwise are harmless can be made hazardous by metabolic deficiencies of the fetus. B) The greatest danger of drug-caused developmental deficits in the fetus is seen in the final trimester. C) Killed-virus vaccines (e.g., tetanus) should not be given during pregnancy, but live-virus vaccines (e.g., measles) are permissible. D) No convincing evidence exists that secondhand smoke is potentially dangerous to the fetus.

A) Both prescription and over-the-counter (OTC) drugs that otherwise are harmless can be made hazardous by metabolic deficiencies of the fetus.

Which statement regarding acronyms in nutrition is accurate? A) Dietary reference intakes (DRIs) consist of recommended dietary allowances (RDAs), adequate intakes (AIs), and upper limits (ULs). B) RDAs are the same as ULs except with better data. C) AIs offer guidelines for avoiding excessive amounts of nutrients. D) They all refer to green leafy vegetables, whole grains, and fruit.

A) Dietary reference intakes (DRIs) consist of recommended dietary allowances (RDAs), adequate intakes (AIs), and upper limits (ULs).

Fibrocystic changes in the breast most often appear in women in their 20s and 30s. The etiology is unknown, but it may be an imbalance of estrogen and progesterone. The nurse who cares for this client should be aware that treatment modalities are conservative. One proven modality that may provide relief is: A) Diuretic administration. B) Including caffeine daily in the diet. C) Increased vitamin C supplementation. D) Application of cold packs to the breast as necessary.

A) Diuretic administration.

The multiple marker test is used to assess the fetus for which condition? A) Down syndrome B) Diaphragmatic hernia C) Congenital cardiac abnormality D) Anencephaly

A) Down syndrome

A benign breast condition that includes dilation and inflammation of the collecting ducts is called: A) Ductal ectasia. B) Intraductal papilloma. C) Chronic cystic disease. D) Fibroadenoma.

A) Ductal ectasia.

A woman complains of severe abdominal and pelvic pain around the time of menstruation that has gotten worse over the last 5 years. She also complains of pain during intercourse and has tried unsuccessfully to get pregnant for the past 18 months. These symptoms are most likely related to: A) Endometriosis. B) PMS. C) Primary dysmenorrhea. D) Secondary dysmenorrhea.

A) Endometriosis.

Which nutrients recommended dietary allowance (RDA) is higher during lactation than during pregnancy? A) Energy (kcal) B) Iron C) Vitamin A D) Folic acid

A) Energy (kcal)

The exact cause of breast cancer remains undetermined. Researchers have found that there are many common risk factors that increase a woman's chance of developing a malignancy. It is essential for the nurse who provides care to women of any age to be aware of which of the following risk factors (Select all that apply)? A) Family history B) Late menarche C) Early menopause D) Race E) Nulliparity or first pregnancy after age 30

A) Family history D) Race E) Nulliparity or first pregnancy after age 30

Examples of sexual risk behaviors associated with exposure to a sexually transmitted infection (STI) include (Select all that apply): A) Fellatio. B) Unprotected anal intercourse. C) Multiple sex partners. D) Dry kissing. E) Abstinence.

A) Fellatio. B) Unprotected anal intercourse. C) Multiple sex partners.

A woman has a thick, white, lumpy, cottage cheese-like discharge, with patches on her labia and in her vagina. She complains of intense pruritus. The nurse practitioner would order which preparation for treatment? A) Fluconazole B) Tetracycline C) Clindamycin D) Acyclovir

A) Fluconazole

Identify a goal of a patient with the following nursing diagnosis: Imbalanced Nutrition: Less Than Body Requirements related to diet choices inadequate to meet nutrient requirements of pregnancy. A) Gain a total of 30 lb. B) Take daily supplements consistently. C) Decrease intake of snack foods. D) Increase intake of complex carbohydrates.

A) Gain a total of 30 lb.

Which viral sexually transmitted infection is characterized by a primary infection followed by recurrent episodes? A) Herpes simplex virus (HSV)-2 B) Human papillomavirus (HPV) C) Human immunodeficiency virus (HIV) D) Cytomegalovirus (CMV)

A) Herpes simplex virus (HSV)-2

As relates to dysfunctional uterine bleeding (DUB), the nurse should be aware that: A) It is most commonly caused by anovulation. B) It most often occurs in middle age. C) The diagnosis of DUB should be the first considered for abnormal menstrual bleeding. D) The most effective medical treatment is steroids.

A) It is most commonly caused by anovulation.

After you complete your nutritional counseling for a pregnant woman, you ask her to repeat your instructions so you can assess her understanding of the instructions given. Which statement indicates that she understands the role of protein in her pregnancy? A) Protein will help my baby grow. B) Eating protein will prevent me from becoming anemic. C) Eating protein will make my baby have strong teeth after he is born. D) Eating protein will prevent me from being diabetic.

A) Protein will help my baby grow.

There is little consensus on the management of premenstrual dysphoric disorder (PMDD). However, nurses can advise women on several self-help modalities that often improve symptoms. The nurse knows that health teaching has been effective when the client reports that she has adopted a number of lifestyle changes, including (Select all that apply): A) Regular exercise. B) Improved nutrition. C) A daily glass of wine. D) Smoking cessation. E) Oil of evening primrose.

A) Regular exercise. B) Improved nutrition. D) Smoking cessation. E) Oil of evening primrose.

A pregnant woman reports that she is still playing tennis at 32 weeks of gestation. The nurse would be most concerned that during and after tennis matches this woman consumes: A) Several glasses of fluid. B) Extra protein sources such as peanut butter. C) Salty foods to replace lost sodium. D) Easily digested sources of carbohydrate.

A) Several glasses of fluid.

Which behavior indicates that a woman is seeking safe passage for herself and her infant? A) She keeps all prenatal appointments B) She eats for two C) She drives her car slowly D) She wears only low-heeled shoes

A) She keeps all prenatal appointments

An essential component of counseling women regarding safe sex practices includes discussion regarding avoiding the exchange of body fluids. The physical barrier promoted for the prevention of sexually transmitted infections and human immunodeficiency virus is the condom. Nurses can help motivate clients to use condoms by initiating a discussion related to a number of aspects of condom use. The most important of these is: A) Strategies to enhance condom use. B) Choice of colors and special features. C) Leaving the decision up to the male partner. D) Places to carry condoms safely.

A) Strategies to enhance condom use.

A woman has just moved to the United States from Mexico. She is 3 months pregnant and has arrived for her first prenatal visit. During her assessment interview, you discover that she has not had any immunizations. Which immunizations should she receive at this point in her pregnancy (Select all that apply)? A) Tetanus B) Diphtheria C) Chickenpox D) Rubella E) Hepatitis B

A) Tetanus B) Diphtheria E) Hepatitis B

What important, immediate postoperative care practice should the nurse remember when caring for a woman who has had a mastectomy? A) The blood pressure (BP) cuff should not be applied to the affected arm. B) Venipuncture for blood work should be performed on the affected arm. C) The affected arm should be used for intravenous (IV) therapy. D) The affected arm should be held down close to the woman's side.

A) The blood pressure (BP) cuff should not be applied to the affected arm.

As relates to the fathers acceptance of the pregnancy and preparation for childbirth, the maternity nurse should know that: A) The father goes through three phases of acceptance of his own. B) The fathers attachment to the fetus cannot be as strong as that of the mother because it does not start until after birth. C) In the last 2 months of pregnancy, most expectant fathers suddenly get very protective of their established lifestyle and resist making changes to the home. D) Typically men remain ambivalent about fatherhood right up to the birth of their child.

A) The father goes through three phases of acceptance of his own.

To help a woman reduce the severity of nausea caused by morning sickness, the nurse might suggest that she: A) Try a tart food or drink such as lemonade or salty foods such as potato chips. B) Drink plenty of fluids early in the day. C) Brush her teeth immediately after eating. D) Never snack before bedtime.

A) Try a tart food or drink such as lemonade or salty foods such as potato chips.

While teaching the expectant mother about personal hygiene during pregnancy, maternity nurses should be aware that: A) Tub bathing is permitted even in late pregnancy unless membranes have ruptured. B) The perineum should be wiped from back to front. C) Bubble bath and bath oils are permissible because they add an extra soothing and cleansing action to the bath. D) Expectant mothers should use specially treated soap to cleanse the nipples.

A) Tub bathing is permitted even in late pregnancy unless membranes have ruptured.

Signs and symptoms that a woman should report immediately to her health care provider include (Select all that apply): A) Vaginal bleeding. B) Rupture of membranes. C) Heartburn accompanied by severe headache. D) Decreased libido. E) Urinary frequency.

A) Vaginal bleeding. B) Rupture of membranes. C) Heartburn accompanied by severe headache.

Because pregnant women may need surgery during pregnancy, nurses should be aware that: a. the diagnosis of appendicitis may be difficult because the normal signs and symptoms mimic some normal changes in pregnancy. b. rupture of the appendix is less likely in pregnant women because of the close monitoring. c. surgery for intestinal obstructions should be delayed as long as possible because it usually affects the pregnancy. d. when pregnancy takes over, a woman is less likely to have ovarian problems that require invasive responses.

ANS: A Both appendicitis and pregnancy are linked with nausea, vomiting, and increased white blood cell count. Rupture of the appendix is two to three times more likely in pregnant women. Surgery to remove obstructions should be done right away. It usually does not affect the pregnancy. Pregnancy predisposes a woman to ovarian problems.

Spontaneous termination of a pregnancy is considered to be an abortion if: a. the pregnancy is less than 20 weeks. b. the fetus weighs less than 1000g. c. the products of conception are passed intact. d. no evidence exists of intrauterine infection.

ANS: A An abortion is the termination of pregnancy before the age of viability (20 weeks). The weight of the fetus is not considered because some older fetuses may have a low birth weight. A spontaneous abortion may be complete or incomplete. A spontaneous abortion may be caused by many problems, one being intrauterine infection.

The priority nursing intervention when admitting a pregnant woman who has experienced a bleeding episode in late pregnancy is to: a. assess fetal heart rate (FHR) and maternal vital signs. b. perform a venipuncture for hemoglobin and hematocrit levels. c. place clean disposable pads to collect any drainage. d. monitor uterine contractions.

ANS: A Assessment of the FHR and maternal vital signs will assist the nurse in determining the degree of the blood loss and its effect on the mother and fetus. The most important assessment is to check mother/fetal well-being. The blood levels can be obtained later. It is important to assess future bleeding; however, the top priority remains mother/fetal well-being. Monitoring uterine contractions is important but not the top priority.

Approximately 10% to 15% of all clinically recognized pregnancies end in miscarriage. Which is the most common cause of spontaneous abortion? a. Chromosomal abnormalities b. Infections c. Endocrine imbalance d. Immunologic factors

ANS: A At least 50% of pregnancy losses result from chromosomal abnormalities that are incompatible with life. Maternal infection may be a cause of early miscarriage. Endocrine imbalances such as hypothyroidism or diabetes are possible causes for early pregnancy loss. Women who have repeated early pregnancy losses appear to have immunologic factors that play a role in spontaneous abortion incidents.

Which order should the nurse expect for a patient admitted with a threatened abortion? a. Bed rest b. Ritodrine IV c. NPO d. Narcotic analgesia every 3 hours, PRN

ANS: A Decreasing the woman's activity level may alleviate the bleeding and allow the pregnancy to continue. Ritodrine IV is not the first drug of choice for tocolytic medications. There is no reason for having the woman placed NPO. At times dehydration may produce contractions, so hydration is important. Narcotic analgesia will not decrease the contractions. It may mask the severity of the contractions.

A woman with severe preeclampsia has been receiving magnesium sulfate by intravenous infusion for 8 hours. The nurse assesses the woman and documents the following findings: temperature of 37.1° C, pulse rate of 96 beats/min, respiratory rate of 24 breaths/min, blood pressure (BP) of 155/112 mm Hg, 3+ deep tendon reflexes, and no ankle clonus. The nurse calls the physician, anticipating an order for: a. hydralazine. b. magnesium sulfate bolus. c. diazepam. d. calcium gluconate.

ANS: A Hydralazine is an antihypertensive commonly used to treat hypertension in severe preeclampsia. Typically, it is administered for a systolic BP greater than 160 mm Hg or a diastolic BP greater than 110 mm Hg. An additional bolus of magnesium sulfate may be ordered for increasing signs of central nervous system irritability related to severe preeclampsia (e.g., clonus) or if eclampsia develops. Diazepam sometimes is used to stop or shorten eclamptic seizures. Calcium gluconate is used as the antidote for magnesium sulfate toxicity. The patient is not currently displaying any signs or symptoms of magnesium toxicity.

In planning care for women with preeclampsia, nurses should be aware that: a. induction of labor is likely, as near term as possible. b. if at home, the woman should be confined to her bed, even with mild preeclampsia. c. a special diet low in protein and salt should be initiated. d. vaginal birth is still an option, even in severe cases.

ANS: A Induction of labor is likely, as near term as possible; however, at less than 37 weeks of gestation, immediate delivery may not be in the best interest of the fetus. Strict bed rest is becoming controversial for mild cases; some women in the hospital are even allowed to move around. Diet and fluid recommendations are much the same as for healthy pregnant women, although some authorities have suggested a diet high in protein. Women with severe preeclampsia should expect a cesarean delivery.

The nurse caring for pregnant women must be aware that the most common medical complication of pregnancy is: a. hypertension. b. hyperemesis gravidarum. c. hemorrhagic complications. d. infections.

ANS: A Preeclampsia and eclampsia are two noted deadly forms of hypertension. A large percentage of pregnant women will have nausea and vomiting, but a relatively few have the severe form called hyperemesis gravidarum. Hemorrhagic complications are the second most common medical complication of pregnancy; hypertension is the most common.

In caring for the woman with disseminated intravascular coagulation (DIC), what order should the nurse anticipate? a. Administration of blood b. Preparation of the patient for invasive hemodynamic monitoring c. Restriction of intravascular fluids d. Administration of steroids

ANS: A Primary medical management in all cases of DIC involves correction of the underlying cause, volume replacement, blood component therapy, optimization of oxygenation and perfusion status, and continued reassessment of laboratory parameters. Central monitoring would not be ordered initially in a patient with DIC because this can contribute to more areas of bleeding. Management of DIC would include volume replacement, not volume restriction. Steroids are not indicated for the management of DIC.

In caring for an immediate after birth patient, you note petechiae and oozing from her IV site. You would monitor her closely for the clotting disorder: a. disseminated intravascular coagulation (DIC). b. amniotic fluid embolism (AFE). c. hemorrhage. d. HELLP syndrome.

ANS: A The diagnosis of DIC is made according to clinical findings and laboratory markers. Physical examination reveals unusual bleeding. Petechiae may appear around a blood pressure cuff on the woman's arm. Excessive bleeding may occur from the site of slight trauma such as venipuncture sites. These symptoms are not associated with AFE, nor is AFE a bleeding disorder. Hemorrhage occurs for a variety of reasons in the after birth patient. These symptoms are associated with DIC. Hemorrhage would be a finding associated with DIC and is not a clotting disorder in and of itself. HELLP is not a clotting disorder, but it may contribute to the clotting disorder DIC.

5. In vitro fertilization-embryo transfer (IVF-ET) is a common approach for women with blocked fallopian tubes or unexplained infertility and for men with very low sperm counts. A husband and wife have arrived for their preprocedural interview. The husband asks the nurse to explain what the procedure entails. The nurse's most appropriate response is: a. "IVF-ET is a type of assisted reproductive therapy that involves collecting eggs from your wife's ovaries, fertilizing them in the laboratory with your sperm, and transferring the embryo to her uterus." b. "A donor embryo will be transferred into your wife's uterus." c. "Donor sperm will be used to inseminate your wife." d. "Don't worry about the technical stuff; that's what we are here for."

ANS: A A woman's eggs are collected from her ovaries, fertilized in the laboratory with sperm, and transferred to her uterus after normal embryonic development has occurred. The statement, "A donor embryo will be transferred into your wife's uterus" describes therapeutic donor insemination. "Donor sperm will be used to inseminate your wife" describes the procedure for a donor embryo. "Don't worry about the technical stuff; that's what we are here for" discredits the patient's need for teaching and is an inappropriate response.

27. Postcoital contraception with Ella: a. requires that the first dose be taken within 72 hours of unprotected intercourse. b. requires that the woman take second and third doses at 24 and 36 hours after the first dose. c. must be taken in conjunction with an IUD insertion. d. is commonly associated with the side effect of menorrhagia.

ANS: A Emergency contraception is most effective when used within 72 hours of intercourse; however, it may be used with lessened effectiveness 120 hours later. Insertion of the copper IUD within 5 days of intercourse may also be used and is up to 99% effective. The most common side effect of postcoital contraception is nausea.

9. A woman has chosen the calendar method of conception control. During the assessment process, it is most important that the nurse: a. obtain a history of menstrual cycle lengths for the past 6 months. b. determine the patient's weight gain and loss pattern for the previous year. c. examine skin pigmentation and hair texture for hormonal changes. d. explore the patient's previous experiences with conception control.

ANS: A The calendar method of conception control is based on the number of days in each cycle, counting from the first day of menses. The fertile period is determined after the lengths of menstrual cycles have been accurately recorded for 6 months. Weight gain or loss may be partly related to hormonal fluctuations, but it has no bearing on use of the calendar method. Integumentary changes may be related to hormonal changes, but they are not indicators for use of the calendar method. Exploring previous experiences with conception control may demonstrate patient understanding and compliancy, but it is not the most important aspect to assess for discussion of the calendar method.

29. A physician prescribes clomiphene citrate (Clomid, Serophene) for a woman experiencing infertility. She is very concerned about the risk of multiple births. The nurse's most appropriate response is: a. "This is a legitimate concern. Would you like to discuss this further before your treatment begins?" b. "No one has ever had more than triplets with Clomid." c. "Ovulation will be monitored with ultrasound so that this will not happen." d. "Ten percent is a very low risk, so you don't need to worry too much."

ANS: A The incidence of multiple pregnancies with the use of these medications is significantly increased. The patient's concern is legitimate and should be discussed so that she can make an informed decision. Stating that no one has ever had "more than triplets" is inaccurate and negates the patient's concerns. Ultrasound cannot ensure that a multiple pregnancy will not occur. The percentage quoted in this statement is inaccurate. The comment "don't worry" discredits the patient's concern.

18. A woman is 16 weeks pregnant and has elected to terminate her pregnancy. The nurse knows that the most common technique used for medical termination of a pregnancy in the second trimester is: a. dilation and evacuation (D&E). b. instillation of hypertonic saline into the uterine cavity. c. intravenous administration of Pitocin. d. vacuum aspiration.

ANS: A The most common technique for medical termination of a pregnancy in the second trimester is D&E. It is usually performed between 13 and 16 weeks. Hypertonic solutions injected directly into the uterus account for less than 1% of all abortions because other methods are safer and easier to use. Intravenous administration of Pitocin is used to induce labor in a woman with a third-trimester fetal demise. Vacuum aspiration is used for abortions in the first trimester.

16. A woman was treated recently for toxic shock syndrome (TSS). She has intercourse occasionally and uses over-the-counter protection. On the basis of her history, what contraceptive method should she and her partner avoid? a. Cervical cap b. Condom c. Vaginal film d. Vaginal sheath

ANS: A Women with a history of TSS should not use a cervical cap. Condoms, vaginal films, and vaginal sheaths are not contraindicated for a woman with a history of TSS.

The reported incidence of ectopic pregnancy in the United States has risen steadily over the past two decades. Causes include the increase in STDs accompanied by tubal infection and damage. The popularity of contraceptive devices such as the IUD has also increased the risk for ectopic pregnancy. The nurse who suspects that a patient has early signs of ectopic pregnancy should be observing her for symptoms such as: (Select all that apply.) a. pelvic pain. b. abdominal pain. c. unanticipated heavy bleeding. d. vaginal spotting or light bleeding. e. missed period.

ANS: A, B, D, E A missed period or spotting can easily be mistaken by the patient as early signs of pregnancy. More subtle signs depend on exactly where the implantation occurs. The nurse must be thorough in her assessment because pain is not a normal symptom of early pregnancy. As the fallopian tube tears open and the embryo is expelled, the patient often exhibits severe pain accompanied by intra-abdominal hemorrhage. This may progress to hypovolemic shock with minimal or even no external bleeding. In about half of women, shoulder and neck pain results from irritation of the diaphragm from the hemorrhage.

A patient who has undergone a dilation and curettage for early pregnancy loss is likely to be discharged the same day. The nurse must ensure that vital signs are stable, bleeding has been controlled, and the woman has adequately recovered from the administration of anesthesia. To promote an optimal recovery, discharge teaching should include: (Select all that apply.) a. iron supplementation. b. resumption of intercourse at 6 weeks following the procedure. c. referral to a support group if necessary. d. expectation of heavy bleeding for at least 2 weeks. e. emphasizing the need for rest.

ANS: A, C, E The woman should be advised to consume a diet high in iron and protein. For many women iron supplementation is also necessary. Acknowledge that the patient has experienced a loss, albeit early. She can be taught to expect mood swings and possibly depression. Referral to a support group, clergy, or professional counseling may be necessary. Discharge teaching should emphasize the need for rest. Nothing should be placed in the vagina for 2 weeks after the procedure. This includes tampons and vaginal intercourse. The purpose of this recommendation is to prevent infection. Should infection occur, antibiotics may be prescribed. The patient should expect a scant, dark discharge for 1 to 2 weeks. Should heavy, profuse, or bright bleeding occur, she should be instructed to contact her provider.

1. You (the nurse) are reviewing the educational packet provided to a patient about tubal ligation. What is an important fact you should point out? (Select all that apply.) a. "It is highly unlikely that you will become pregnant after the procedure." b. "This is an effective form of 100% permanent sterilization. You won't be able to get pregnant." c. "Sterilization offers some form of protection against sexually transmitted infections (STIs)." d. "Sterilization offers no protection against STIs." e. "Your menstrual cycle will greatly increase after your sterilization."

ANS: A, D A woman is unlikely to become pregnant after tubal ligation, although it is not 100% effective. Sterilization offers no protection against STIs. The menstrual cycle typically remains the same after a tubal ligation.

The patient that you are caring for has severe preeclampsia and is receiving a magnesium sulfate infusion. You become concerned after assessment when the woman exhibits: a. a sleepy, sedated affect. b. a respiratory rate of 10 breaths/min. c. deep tendon reflexes of 2. d. absent ankle clonus.

ANS: B A respiratory rate of 10 breaths/min indicates that the patient is experiencing respiratory depression from magnesium toxicity. Because magnesium sulfate is a central nervous system depressant, the patient will most likely become sedated when the infusion is initiated. Deep tendon reflexes of two and absent ankle clonus are normal findings.

What laboratory marker is indicative of disseminated intravascular coagulation (DIC)? a. Bleeding time of 10 minutes b. Presence of fibrin split products c. Thrombocytopenia d. Hyperfibrinogenemia

ANS: B Degradation of fibrin leads to the accumulation of fibrin split products in the blood. Bleeding time in DIC is normal. Low platelets may occur with but are not indicative of DIC because they may result from other coagulopathies. Hypofibrinogenemia would occur with DIC.

A patient with pregnancy-induced hypertension is admitted complaining of pounding headache, visual changes, and epigastric pain. Nursing care is based on the knowledge that these signs are an indication of: a. anxiety due to hospitalization. b. worsening disease and impending convulsion. c. effects of magnesium sulfate. d. gastrointestinal upset.

ANS: B Headache and visual disturbances are caused by increased cerebral edema. Epigastric pain indicates distention of the hepatic capsules and often warns that a convulsion is imminent. These are danger signs showing increased cerebral edema and impending convulsion and should be treated immediately. The patient has not been started on magnesium sulfate treatment yet. Also, these are not anticipated effects of the medication.

Which maternal condition always necessitates delivery by cesarean section? a. Partial abruptio placentae b. Total placenta previa c. Ectopic pregnancy d. Eclampsia

ANS: B In total placenta previa, the placenta completely covers the cervical os. The fetus would die if a vaginal delivery occurred. If the mother has stable vital signs and the fetus is alive, a vaginal delivery can be attempted in cases of partial abruptio placentae. If the fetus has died, a vaginal delivery is preferred. The most common ectopic pregnancy is a tubal pregnancy, which is usually detected and treated in the first trimester. Labor can be safely induced if the eclampsia is under control.

The nurse caring for a woman hospitalized for hyperemesis gravidarum should expect that initial treatment to involve: a. corticosteroids to reduce inflammation. b. IV therapy to correct fluid and electrolyte imbalances. c. an antiemetic, such as pyridoxine, to control nausea and vomiting. d. enteral nutrition to correct nutritional deficits.

ANS: B Initially, the woman who is unable to keep down clear liquids by mouth requires IV therapy for correction of fluid and electrolyte imbalances. Corticosteroids have been used successfully to treat refractory hyperemesis gravidarum; however, they are not the expected initial treatment for this disorder. Pyridoxine is vitamin B6, not an antiemetic. Promethazine, a common antiemetic, may be prescribed. In severe cases of hyperemesis gravidarum, enteral nutrition via a feeding tube may be necessary to correct maternal nutritional deprivation. This is not an initial treatment for this patient.

An abortion in which the fetus dies but is retained within the uterus is called a(n): a. inevitable abortion. b. missed abortion. c. incomplete abortion. d. threatened abortion.

ANS: B Missed abortion refers to retention of a dead fetus in the uterus. An inevitable abortion means that the cervix is dilating with the contractions. An incomplete abortion means that not all of the products of conception were expelled. With a threatened abortion the woman has cramping and bleeding but not cervical dilation.

The most prevalent clinical manifestation of abruptio placentae (as opposed to placenta previa) is: a. bleeding. b. intense abdominal pain. c. uterine activity. d. cramping.

ANS: B Pain is absent with placenta previa and may be agonizing with abruptio placentae. Bleeding may be present in varying degrees for both placental conditions. Uterine activity and cramping may be present with both placental conditions.

A 26-year-old pregnant woman, gravida 2, para 1-0-0-1 is 28 weeks pregnant when she experiences bright red, painless vaginal bleeding. On her arrival at the hospital, what would be an expected diagnostic procedure? a. Amniocentesis for fetal lung maturity b. Ultrasound for placental location c. Contraction stress test (CST) d. Internal fetal monitoring

ANS: B The presence of painless bleeding should always alert the health care team to the possibility of placenta previa. This can be confirmed through ultrasonography. Amniocentesis would not be performed on a woman who is experiencing bleeding. In the event of an imminent delivery, the fetus would be presumed to have immature lungs at this gestational age, and the mother would be given corticosteroids to aid in fetal lung maturity. A CST would not be performed at a preterm gestational age. Furthermore, bleeding would be a contraindication to this test. Internal fetal monitoring would be contraindicated in the presence of bleeding.

The perinatal nurse is giving discharge instructions to a woman after suction curettage secondary to a hydatidiform mole. The woman asks why she must take oral contraceptives for the next 12 months. The best response from the nurse would be: a. "If you get pregnant within 1 year, the chance of a successful pregnancy is very small. Therefore, if you desire a future pregnancy, it would be better for you to use the most reliable method of contraception available." b. "The major risk to you after a molar pregnancy is a type of cancer that can be diagnosed only by measuring the same hormone that your body produces during pregnancy. If you were to get pregnant, it would make the diagnosis of this cancer more difficult." c. "If you can avoid a pregnancy for the next year, the chance of developing a second molar pregnancy is rare. Therefore, to improve your chance of a successful pregnancy, it is better not to get pregnant at this time." d. "Oral contraceptives are the only form of birth control that will prevent a recurrence of a molar pregnancy."

ANS: B This is an accurate statement. Beta-human chorionic gonadotropin (hCG) levels will be drawn for 1 year to ensure that the mole is completely gone. There is an increased chance of developing choriocarcinoma after the development of a hydatidiform mole. The goal is to achieve a "zero" hCG level. If the woman were to become pregnant, it could obscure the presence of the potentially carcinogenic cells. Women should be instructed to use birth control for 1 year after treatment for a hydatidiform mole. The rationale for avoiding pregnancy for 1 year is to ensure that carcinogenic cells are not present. Any contraceptive method except an intrauterine device is acceptable.

A laboring woman with no known risk factors suddenly experiences spontaneous rupture of membranes (ROM). The fluid consists of bright red blood. Her contractions are consistent with her current stage of labor. There is no change in uterine resting tone. The fetal heart rate begins to decline rapidly after the ROM. The nurse should suspect the possibility of: a. placenta previa. b. vasa previa. c. severe abruptio placentae. d. disseminated intravascular coagulation (DIC).

ANS: B Vasa previa is the result of a velamentous insertion of the umbilical cord. The umbilical vessels are not surrounded by Wharton jelly and have no supportive tissue. They are at risk for laceration at any time, but laceration occurs most frequently during ROM. The sudden appearance of bright red blood at the time of ROM and a sudden change in the fetal heart rate without other known risk factors should immediately alert the nurse to the possibility of vasa previa. The presence of placenta previa most likely would be ascertained before labor and would be considered a risk factor for this pregnancy. In addition, if the woman had a placenta previa, it is unlikely that she would be allowed to pursue labor and a vaginal birth. With the presence of severe abruptio placentae, the uterine tonicity would typically be tetanus (i.e., a board-like uterus). DIC is a pathologic form of diffuse clotting that consumes large amounts of clotting factors and causes widespread external bleeding, internal bleeding, or both. DIC is always a secondary diagnosis, often associated with obstetric risk factors such as HELLP syndrome. This woman did not have any prior risk factors.

Women with hyperemesis gravidarum: a. are a majority because 80% of all pregnant women suffer from it at some time. b. have vomiting severe and persistent enough to cause weight loss, dehydration, and electrolyte imbalance. c. need intravenous (IV) fluid and nutrition for most of their pregnancy. d. often inspire similar, milder symptoms in their male partners and mothers.

ANS: B Women with hyperemesis gravidarum have severe vomiting; however, treatment for several days sets things right in most cases. Although 80% of pregnant women experience nausea and vomiting, fewer than 1% (0.5%) proceed to this severe level. IV administration may be used at first to restore fluid levels, but it is seldom needed for very long. Women suffering from this condition want sympathy because some authorities believe that difficult relationships with mothers and/or partners may be the cause.

23. Which contraceptive method best protects against sexually transmitted infections (STIs) and human immunodeficiency virus (HIV)? a. Periodic abstinence b. Barrier methods c. Hormonal methods d. They all offer about the same protection.

ANS: B Barrier methods such as condoms best protect against STIs and HIV. Periodic abstinence and hormonal methods (the pill) offer no protection against STIs or HIV.

13. A woman who has a seizure disorder and takes barbiturates and phenytoin sodium daily asks the nurse about the pill as a contraceptive choice. The nurse's most appropriate response would be: a. "This is a highly effective method, but it has some side effects." b. "Your current medications will reduce the effectiveness of the pill." c. "The pill will reduce the effectiveness of your seizure medication." d. "This is a good choice for a woman of your age and personal history."

ANS: B Because the liver metabolizes oral contraceptives, their effectiveness is reduced when they are taken simultaneously with anticonvulsants. The statement "Your current medications will reduce the effectiveness of the pill" is true, but it is not the most appropriate response. The anticonvulsant will reduce the effectiveness of the pill, not the other way around. The statement "This is a good choice for a woman of your age and personal history" does not teach the patient that the effectiveness of the pill may be reduced because of her anticonvulsant therapy.

17. An unmarried young woman describes her sex life as "active" and involving "many" partners. She wants a contraceptive method that is reliable and does not interfere with sex. She requests an intrauterine device (IUD). The nurse's most appropriate response is: a. "The IUD does not interfere with sex." b. "The risk of pelvic inflammatory disease (PID) will be higher for you." c. "The IUD will protect you from sexually transmitted infections (STIs)." d. "Pregnancy rates are high with IUDs."

ANS: B Disadvantages of IUDs include an increased risk of PID in the first 20 days after insertion and the risks of bacterial vaginosis and uterine perforation. The IUD offers no protection against STIs or human immunodeficiency virus. Because this woman has multiple sex partners, she is at higher risk of developing a STI. The IUD does not protect against infection, as does a barrier method. Although the statement "The IUD does not interfere with sex" may be correct, it is not the most appropriate response. The IUD offers no protection from STIs. The typical failure rate of the IUD in the first year of use is 0.8%.

11. A married couple is discussing alternatives for pregnancy prevention and has asked about fertility awareness methods (FAMs). The nurse's most appropriate reply is: a. "They're not very effective, and it's very likely you'll get pregnant." b. "They can be effective for many couples, but they require motivation." c. "These methods have a few advantages and several health risks." d. "You would be much safer going on the pill and not having to worry."

ANS: B FAMs are effective with proper vigilance about ovulatory changes in the body and adherence to coitus intervals. They are effective if used correctly by a woman with a regular menstrual cycle. The typical failure rate for all FAMs is 25% during the first year of use. FAMs have no associated health risks. The use of birth control has associated health risks. In addition, taking a pill daily requires compliance on the patient's part.

6. With regard to the assessment of female, male, and couple infertility, nurses should be aware that: a. the couple's religious, cultural, and ethnic backgrounds provide emotional clutter that does not affect the clinical scientific diagnosis. b. the investigation takes 3 to 4 months and a significant financial investment. c. the woman is assessed first; if she is not the problem, the male partner is analyzed. d. semen analysis is for men; the postcoital test is for women.

ANS: B Fertility assessment and diagnosis take time, money, and commitment from the couple. Religious, cultural, and ethnic-bred attitudes about fertility and related issues always have an impact on diagnosis and assessment. Both partners are assessed systematically and simultaneously, as individuals and as a couple. Semen analysis is for men, but the postcoital test is for the couple.

4. A woman enquires about herbal alternative methods for improving fertility. Which statement by the nurse is the most appropriate when instructing the patient in which herbal preparations to avoid while trying to conceive? a. "You should avoid nettle leaf, dong quai, and vitamin E while you are trying to get pregnant." b. "You may want to avoid licorice root, lavender, fennel, sage, and thyme while you are trying to conceive." c. "You should not take anything with vitamin E, calcium, or magnesium. They will make you infertile." d. "Herbs have no bearing on fertility."

ANS: B Herbs that a woman should avoid while trying to conceive include licorice root, yarrow, wormwood, ephedra, fennel, golden seal, lavender, juniper, flaxseed, pennyroyal, passionflower, wild cherry, cascara, sage, thyme, and periwinkle. Nettle leaf, dong quai, and vitamin E all promote fertility. Vitamin E, calcium, and magnesium may promote fertility and conception. All supplements and herbs should be purchased from trusted sources.

12. A male patient asks the nurse why it is better to purchase condoms that are not lubricated with nonoxynol-9 (a common spermicide). The nurse's most appropriate response is: a. "The lubricant prevents vaginal irritation." b. "It has also been linked to an increase in the transmission of human immunodeficiency virus." c. "The additional lubrication improves sex." d. "Nonoxynol-9 improves penile sensitivity."

ANS: B The statement "Nonoxynol-9 does not provide protection against sexually transmitted infections, as originally thought; it has also been linked to an increase in the transmission of human immunodeficiency virus and can cause genital lesions" is true. Nonoxynol-9 may cause vaginal irritation, has no effect on the quality of sexual activity, and has no effect on penile sensitivity.

25. With regard to the use of intrauterine devices (IUDs), nurses should be aware that: a. return to fertility can take several weeks after the device is removed. b. IUDs containing copper can provide an emergency contraception option if inserted within a few days of unprotected intercourse. c. IUDs offer the same protection against sexually transmitted infections (STIs) as the diaphragm. d. consent forms are not needed for IUD insertion.

ANS: B The woman has up to 5 days to insert the IUD after unprotected sex. Return to fertility is immediate after removal of the IUD. IUDs offer no protection for STIs. A consent form is required for insertion, as is a negative pregnancy test.

A woman presents to the emergency department with complaints of bleeding and cramping. The initial nursing history is significant for a last menstrual period 6 weeks ago. On sterile speculum examination, the primary care provider finds that the cervix is closed. The anticipated plan of care for this woman would be based on a probable diagnosis of which type of spontaneous abortion? a. Incomplete b. Inevitable c. Threatened d. Septic

ANS: C A woman with a threatened abortion presents with spotting, mild cramps, and no cervical dilation. A woman with an incomplete abortion would present with heavy bleeding, mild to severe cramping, and cervical dilation. An inevitable abortion manifests with the same symptoms as an incomplete abortion: heavy bleeding, mild to severe cramping, and cervical dilation. A woman with a septic abortion presents with malodorous bleeding and typically a dilated cervix.

Which condition indicates concealed hemorrhage when the patient experiences an abruptio placentae? a. Decrease in abdominal pain b. Bradycardia c. Hard, board-like abdomen d. Decrease in fundal height

ANS: C Concealed hemorrhage occurs when the edges of the placenta do not separate. The formation of a hematoma behind the placenta and subsequent infiltration of the blood into the uterine muscle results in a very firm, board-like abdomen. Abdominal pain may increase. The patient will have shock symptoms that include tachycardia. As bleeding occurs, the fundal height will increase.

A woman arrives for evaluation of her symptoms, which include a missed period, adnexal fullness, tenderness, and dark red vaginal bleeding. On examination the nurse notices an ecchymotic blueness around the woman's umbilicus and recognizes this assessment finding as: a. normal integumentary changes associated with pregnancy. b. Turner's sign associated with appendicitis. c. Cullen's sign associated with a ruptured ectopic pregnancy. d. Chadwick's sign associated with early pregnancy.

ANS: C Cullen's sign, the blue ecchymosis seen in the umbilical area, indicates hematoperitoneum associated with an undiagnosed ruptured intraabdominal ectopic pregnancy. Linea nigra on the abdomen is the normal integumentary change associated with pregnancy. It manifests as a brown, pigmented, vertical line on the lower abdomen. Turner's sign is ecchymosis in the flank area, often associated with pancreatitis. Chadwick's sign is the blue-purple color of the cervix that may be seen during or around the eighth week of pregnancy.

The labor of a pregnant woman with preeclampsia is going to be induced. Before initiating the Pitocin infusion, the nurse reviews the woman's latest laboratory test findings, which reveal a platelet count of 90,000, an elevated aspartate transaminase (AST) level, and a falling hematocrit. The nurse notifies the physician because the laboratory results are indicative of: a. eclampsia. b. disseminated intravascular coagulation (DIC). c. HELLP syndrome. d. idiopathic thrombocytopenia.

ANS: C HELLP syndrome is a laboratory diagnosis for a variant of severe preeclampsia that involves hepatic dysfunction characterized by hemolysis (H), elevated liver enzymes (EL), and low platelets (LP). Eclampsia is determined by the presence of seizures. DIC is a potential complication associated with HELLP syndrome. Idiopathic thrombocytopenia is the presence of low platelets of unknown cause and is not associated with preeclampsia.

In providing nutritional counseling for the pregnant woman experiencing cholecystitis, the nurse would: a. assess the woman's dietary history for adequate calories and proteins. b. instruct the woman that the bulk of calories should come from proteins. c. instruct the woman to eat a low-fat diet and avoid fried foods. d. instruct the woman to eat a low-cholesterol, low-salt diet.

ANS: C Instructing the woman to eat a low-fat diet and avoid fried foods is appropriate nutritional counseling for this patient. Caloric and protein intake do not predispose a woman to the development of cholecystitis. The woman should be instructed to limit protein intake and choose foods that are high in carbohydrates. A low-cholesterol diet may be the result of limiting fats. However, a low-salt diet is not indicated.

Magnesium sulfate is given to women with preeclampsia and eclampsia to: a. improve patellar reflexes and increase respiratory efficiency. b. shorten the duration of labor. c. prevent and treat convulsions. d. prevent a boggy uterus and lessen lochial flow.

ANS: C Magnesium sulfate is the drug of choice to prevent convulsions, although it can generate other problems. Loss of patellar reflexes and respiratory depression are signs of magnesium toxicity. Magnesium sulfate can increase the duration of labor. Women are at risk for a boggy uterus and heavy lochial flow as a result of magnesium sulfate therapy.

Methotrexate is recommended as part of the treatment plan for which obstetric complication? a. Complete hydatidiform mole b. Missed abortion c. Unruptured ectopic pregnancy d. Abruptio placentae

ANS: C Methotrexate is an effective, nonsurgical treatment option for a hemodynamically stable woman whose ectopic pregnancy is unruptured and less than 4 cm in diameter. Methotrexate is not indicated or recommended as a treatment option for complete hydatidiform mole, missed abortion, and abruptio placentae.

A primigravida is being monitored in her prenatal clinic for preeclampsia. What finding should concern her nurse? a. Blood pressure (BP) increase to 138/86 mm Hg. b. Weight gain of 0.5 kg during the past 2 weeks. c. A dipstick value of 3+ for protein in her urine. d. Pitting pedal edema at the end of the day.

ANS: C Proteinuria is defined as a concentration of 1+ or greater via dipstick measurement. A dipstick value of 3+ should alert the nurse that additional testing or assessment should be made. Generally, hypertension is defined as a BP of 140/90 or an increase in systolic pressure of 30 mm Hg or in diastolic pressure of 15 mm Hg. Preeclampsia may be manifested as a rapid weight gain of more than 2 kg in 1 week. Edema occurs in many normal pregnancies and in women with preeclampsia. Therefore, the presence of edema is no longer considered diagnostic of preeclampsia.

Which condition would not be classified as a bleeding disorder in late pregnancy? a. Placenta previa b. Abruptio placentae c. Spontaneous abortion d. Cord insertion

ANS: C Spontaneous abortion is another name for miscarriage; by definition it occurs early in pregnancy. Placenta previa is a cause of bleeding disorders in later pregnancy. Abruptio placentae is a cause of bleeding disorders in later pregnancy. Cord insertion is a cause of bleeding disorders in later pregnancy.

Nurses should be aware that HELLP syndrome: a. is a mild form of preeclampsia. b. can be diagnosed by a nurse alert to its symptoms. c. is characterized by hemolysis, elevated liver enzymes, and low platelets. d. is associated with preterm labor but not perinatal mortality.

ANS: C The acronym HELLP stands for hemolysis (H), elevated liver enzymes (EL), and low platelets (LP). HELLP syndrome is a variant of severe preeclampsia. HELLP syndrome is difficult to identify because the symptoms often are not obvious. It must be diagnosed in the laboratory. Preterm labor is greatly increased, and so is perinatal mortality.

A pregnant woman has been receiving a magnesium sulfate infusion for treatment of severe preeclampsia for 24 hours. On assessment the nurse finds the following vital signs: temperature of 37.3° C, pulse rate of 88 beats/min, respiratory rate of 10 breaths/min, blood pressure (BP) of 148/90 mm Hg, absent deep tendon reflexes, and no ankle clonus. The patient complains, "I'm so thirsty and warm." The nurse: a. calls for a stat magnesium sulfate level. b. administers oxygen. c. discontinues the magnesium sulfate infusion. d. prepares to administer hydralazine.

ANS: C The patient is displaying clinical signs and symptoms of magnesium toxicity. Magnesium should be discontinued immediately. In addition, calcium gluconate, the antidote for magnesium, may be administered. Hydralazine is an antihypertensive commonly used to treat hypertension in severe preeclampsia. Typically it is administered for a systolic BP greater than 160 mm Hg or a diastolic BP greater than 110 mm Hg.

14. Injectable progestins (DMPA, Depo-Provera) are a good contraceptive choice for women who: a. want menstrual regularity and predictability. b. have a history of thrombotic problems or breast cancer. c. have difficulty remembering to take oral contraceptives daily. d. are homeless or mobile and rarely receive health care.

ANS: C Advantages of DMPA include a contraceptive effectiveness comparable to that of combined oral contraceptives with the requirement of only four injections a year. Disadvantages of injectable progestins are prolonged amenorrhea and uterine bleeding. Use of injectable progestin carries an increased risk of venous thrombosis and thromboembolism. To be effective, DMPA injections must be administered every 11 to 13 weeks. Access to health care is necessary to prevent pregnancy or potential complications.

22. While instructing a couple regarding birth control, the nurse should be aware that the method called natural family planning: a. is the same as coitus interruptus, or "pulling out." b. uses the calendar method to align the woman's cycle with the natural phases of the moon. c. is the only contraceptive practice acceptable to the Roman Catholic Church. d. relies on barrier methods during fertility phases.

ANS: C Natural family planning is another name for periodic abstinence, which is the accepted way to pass safely through the fertility phases without relying on chemical or physical barriers. Natural family planning is the only contraceptive practice acceptable to the Roman Catholic Church. "Pulling out" is not the same as periodic abstinence, another name for natural family planning. The phases of the moon are not part of the calendar method or any method.

2. A couple comes in for an infertility workup, having attempted to get pregnant for 2 years. The woman, 37 years, has always had irregular menstrual cycles but is otherwise healthy. The man has fathered two children from a previous marriage and had a vasectomy reversal 2 years ago. The man has had two normal semen analyses, but the sperm seem to be clumped together. What additional test is needed? a. Testicular biopsy b. Antisperm antibodies c. Follicle-stimulating hormone (FSH) level d. Examination for testicular infection

ANS: C The woman has irregular menstrual cycles. The scenario does not indicate that she has had any testing related to this irregularity. Hormone analysis is performed to assess endocrine function of the hypothalamic-pituitary-ovarian axis when menstrual cycles are absent or irregular. Determination of blood levels of prolactin, FSH, luteinizing hormone (LH), estradiol, progesterone, and thyroid hormones may be necessary to diagnose the cause of irregular menstrual cycles. A testicular biopsy would be indicated only in cases of azoospermia (no sperm cells) or severe oligospermia (low number of sperm cells). Antisperm antibodies are produced by a man against his own sperms. This is unlikely to be the case here because the man has already produced children. Examination for testicular infection would be done before semen analysis. Infection would affect spermatogenesis

19. A woman will be taking oral contraceptives using a 28-day pack. The nurse should advise this woman to protect against pregnancy by: a. limiting sexual contact for one cycle after starting the pill. b. using condoms and foam instead of the pill for as long as she takes an antibiotic. c. taking one pill at the same time every day. d. throwing away the pack and using a backup method if she misses two pills during Week 1 of her cycle.

ANS: C To maintain adequate hormone levels for contraception and to enhance compliance, patients should take oral contraceptives at the same time each day. If contraceptives are to be started at any time other than during normal menses or within 3 weeks after birth or abortion, another method of contraception should be used through the first week to prevent the risk of pregnancy. Taken exactly as directed, oral contraceptives prevent ovulation, and pregnancy cannot occur. No strong pharmacokinetic evidence indicates a link between the use of broad-spectrum antibiotics and altered hormone levels in oral contraceptive users. If the patient misses two pills during Week 1, she should take two pills a day for 2 days, finish the package, and use a backup method the next 7 consecutive days.

10. A woman is using the basal body temperature (BBT) method of contraception. She calls the clinic and tells the nurse, "My period is due in a few days, and my temperature has not gone up." The nurse's most appropriate response is: a. "This probably means that you're pregnant." b. "Don't worry; it's probably nothing." c. "Have you been sick this month?" d. "You probably didn't ovulate during this cycle."

ANS: D The absence of a temperature decrease most likely is the result of lack of ovulation. Pregnancy cannot occur without ovulation (which is being measured using the BBT method). A comment such as "Don't worry; it's probably nothing" discredits the patient's concerns. Illness would most likely cause an increase in BBT.

20. A woman had unprotected intercourse 36 hours ago and is concerned that she may become pregnant because it is her "fertile" time. She asks the nurse about emergency contraception. The nurse tells her that: a. it is too late; she needed to begin treatment within 24 hours after intercourse. b. Preven, an emergency contraceptive method, is 98% effective at preventing pregnancy. c. an over-the-counter antiemetic can be taken 1 hour before each contraceptive dose to prevent nausea and vomiting. d. the most effective approach is to use a progestin-only preparation.

ANS: C To minimize the side effect of nausea that occurs with high doses of estrogen and progestin, the woman can take an over-the-counter antiemetic 1 hour before each dose. Emergency contraception is used within 72 hours of unprotected intercourse to prevent pregnancy. Postcoital contraceptive use is 74% to 90% effective at preventing pregnancy. Oral emergency contraceptive regimens may include progestin-only and estrogen-progestin pills. Women with contraindications to estrogen use should use progestin-only pills.

3. A couple is trying to cope with an infertility problem. They want to know what they can do to preserve their emotional equilibrium. The nurse's most appropriate response is: a. "Tell your friends and family so they can help you." b. "Talk only to other friends who are infertile because only they can help." c. "Get involved with a support group. I'll give you some names." d. "Start adoption proceedings immediately because it is very difficult to obtain an infant."

ANS: C Venting negative feelings may unburden the couple. A support group may provide a safe haven for the couple to share their experiences and gain insight from others' experiences. Although talking about their feelings may unburden them of negative feelings, infertility can be a major stressor that affects the couple's relationships with family and friends. Limiting their interactions to other infertile couples may be a beginning point for addressing psychosocial needs, but depending on where the other couple is in their own recovery process, this may or may not help them. The statement about adoption proceedings is not supportive of the psychosocial needs of this couple and may be detrimental to their well-being.

A placenta previa in which the placental edge just reaches the internal os is more commonly known as: a. total. b. partial. c. complete. d. marginal.

ANS: D A placenta previa that does not cover any part of the cervix is termed marginal. With a total placenta previa, the placenta completely covers the os. When the patient experiences a partial placenta previa, the lower border of the placenta is within 3 cm of the internal cervical os but does not completely cover the os. A complete placenta previa is termed total. The placenta completely covers the internal cervical os.

As related to the care of the patient with miscarriage, nurses should be aware that: a. it is a natural pregnancy loss before labor begins. b. it occurs in fewer than 5% of all clinically recognized pregnancies. c. it often can be attributed to careless maternal behavior such as poor nutrition or excessive exercise. d. if it occurs before the 12th week of pregnancy, it may manifest only as moderate discomfort and blood loss.

ANS: D Before the sixth week the only evidence may be a heavy menstrual flow. After the 12th week more severe pain, similar to that of labor, is likely. Miscarriage is a natural pregnancy loss, but by definition it occurs before 20 weeks of gestation, before the fetus is viable. Miscarriages occur in approximately 10% to 15% of all clinically recognized pregnancies. Miscarriage can be caused by a number of disorders or illnesses outside of the mother's control or knowledge.

Nurses should be aware that chronic hypertension: a. is defined as hypertension that begins during pregnancy and lasts for the duration of pregnancy. b. is considered severe when the systolic blood pressure (BP) is greater than 140 mm Hg or the diastolic BP is greater than 90 mm Hg. c. is general hypertension plus proteinuria. d. can occur independently of or simultaneously with gestational hypertension.

ANS: D Hypertension is present before pregnancy or diagnosed before 20 weeks of gestation and persists longer than 6 weeks after birth. The range for hypertension is systolic BP greater than 140 mm Hg or diastolic BP greater than 90 mm Hg. It becomes severe with a diastolic BP of 110 mm Hg or higher. Proteinuria is an excessive concentration of protein in the urine. It is a complication of hypertension, not a defining characteristic.

A woman with preeclampsia has a seizure. The nurse's primary duty during the seizure is to: a. insert an oral airway. b. suction the mouth to prevent aspiration. c. administer oxygen by mask. d. stay with the patient and call for help.

ANS: D If a patient becomes eclamptic, the nurse should stay her and call for help. Insertion of an oral airway during seizure activity is no longer the standard of care. The nurse should attempt to keep the airway patent by turning the patient's head to the side to prevent aspiration. Once the seizure has ended, it may be necessary to suction the patient's mouth. Oxygen would be administered after the convulsion has ended.

A 32-year-old primigravida is admitted with a diagnosis of ectopic pregnancy. Nursing care is based on the knowledge that: a. bed rest and analgesics are the recommended treatment. b. she will be unable to conceive in the future. c. a D&C will be performed to remove the products of conception. d. hemorrhage is the major concern.

ANS: D Severe bleeding occurs if the fallopian tube ruptures. The recommended treatment is to remove the pregnancy before rupture in order to prevent hemorrhaging. If the tube must be removed, the woman's fertility will decrease; however, she will not be infertile. D&C is performed on the inside of the uterine cavity. The ectopic pregnancy is located within the tubes.

A woman at 39 weeks of gestation with a history of preeclampsia is admitted to the labor and birth unit. She suddenly experiences increased contraction frequency of every 1 to 2 minutes; dark red vaginal bleeding; and a tense, painful abdomen. The nurse suspects the onset of: a. eclamptic seizure. b. rupture of the uterus. c. placenta previa. d. placental abruption.

ANS: D Uterine tenderness in the presence of increasing tone may be the earliest finding of premature separation of the placenta (abruptio placentae or placental abruption). Women with hypertension are at increased risk for an abruption. Eclamptic seizures are evidenced by the presence of generalized tonic-clonic convulsions. Uterine rupture manifests as hypotonic uterine activity, signs of hypovolemia, and in many cases the absence of pain. Placenta previa manifests with bright red, painless vaginal bleeding.

8. Although remarkable developments have occurred in reproductive medicine, assisted reproductive therapies are associated with numerous legal and ethical issues. Nurses can provide accurate information about the risks and benefits of treatment alternatives so couples can make informed decisions about their choice of treatment. Which issue would not need to be addressed by an infertile couple before treatment? a. Risks of multiple gestation. b. Whether or how to disclose the facts of conception to offspring. c. Freezing embryos for later use. d. Financial ability to cover the cost of treatment.

ANS: D Although the method of payment is important, obtaining this information is not the responsibility of the nurse. Many states have mandated some form of insurance to assist couples with coverage for infertility. Risk of multiple gestation is a risk of treatment of which the couple needs to be aware. To minimize the chance of multiple gestation, generally only three or fewer embryos are transferred. The couple should be informed that there may be a need for multifetal reduction. Nurses can provide anticipatory guidance on this matter. Depending on the therapy chosen, there may be a need for donor oocytes, sperm, embryos, or a surrogate mother. Couples who have excess embryos frozen for later transfer must be fully informed before consenting to the procedure. A decision must be made regarding the disposal of embryos in the event of death or divorce or if the couple no longer wants the embryos at a future time.

21. Which statement is true about the term contraceptive failure rate? a. It refers to the percentage of users expected to have an accidental pregnancy over a 5-year span. b. It refers to the minimum level that must be achieved to receive a government license. c. It increases over time as couples become more careless. d. It varies from couple to couple, depending on the method and the users.

ANS: D Contraceptive effectiveness varies from couple to couple, depending on how well a contraceptive method is used and how well it suits the couple. The contraceptive failure rate measures the likelihood of accidental pregnancy in the first year only. Failure rates decline over time because users gain experience.

26. Which of the following statements is the most complete and accurate description of medical abortions? a. They are performed only for maternal health. b. They can be achieved through surgical procedures or with drugs. c. They are mostly performed in the second trimester. d. They can be either elective or therapeutic.

ANS: D Medical abortions are performed through the use of medications (rather than surgical procedures). They are mostly done in the first trimester, and they can be either elective (the woman's choice) or therapeutic (for reasons of maternal or fetal health).

7. In their role of implementing a plan of care for infertile couples, nurses should: a. be comfortable with their sexuality and nonjudgmental about others to counsel their patients effectively. b. know about such nonmedical remedies as diet, exercise, and stress management. c. be able to direct patients to sources of information about what herbs to take that might help and which ones to avoid. d. do all of the above plus be knowledgeable about potential drug and surgical remedies.

ANS: D Nurses should be open to and ready to help with a variety of medical and nonmedical approaches.

15. A woman currently uses a diaphragm and spermicide for contraception. She asks the nurse what the major differences are between the cervical cap and the diaphragm. The nurse's most appropriate response is: a. "No spermicide is used with the cervical cap, so it's less messy." b. "The diaphragm can be left in place longer after intercourse." c. "Repeated intercourse with the diaphragm is more convenient." d. "The cervical cap can safely be used for repeated acts of intercourse without adding more spermicide later."

ANS: D The cervical cap can be inserted hours before sexual intercourse without the need for additional spermicide later. No additional spermicide is required for repeated acts of intercourse. Spermicide should be used inside the cap as an additional chemical barrier. The cervical cap should remain in place for 6 hours after the last act of intercourse. Repeated intercourse with the cervical cap is more convenient because no additional spermicide is needed.

24. With regard to the noncontraceptive medical effects of combined oral contraceptive pills (COCs), nurses should be aware that: a. COCs can cause toxic shock syndrome if the prescription is wrong. b. hormonal withdrawal bleeding usually is a bit more profuse than in normal menstruation and lasts a week. c. COCs increase the risk of endometrial and ovarian cancer. d. the effectiveness of COCs can be altered by some over-the-counter medications and herbal supplements.

ANS: D The effectiveness of COCs can be altered by some over-the-counter medications and herbal supplements. Toxic shock syndrome can occur in some diaphragm users, but it is not a consequence of taking oral contraceptive pills. Hormonal withdrawal bleeding usually is lighter than in normal menstruation and lasts a couple of days. Oral contraceptive pills offer protection against the risk of endometrial and ovarian cancers.

28. Informed consent concerning contraceptive use is important because some of the methods: a. are invasive procedures that require hospitalization. b. require a surgical procedure to insert. c. may not be reliable. d. have potentially dangerous side effects.

ANS: D To make an informed decision about the use of contraceptives, it is important for couples to be aware of potential side effects. The only contraceptive method that is a surgical procedure and requires hospitalization is sterilization. Some methods have greater efficacy than others, and this should be included in the teaching.

1. A man smokes two packs of cigarettes a day. He wants to know if smoking is contributing to the difficulty he and his wife are having getting pregnant. The nurse's most appropriate response is: a. "Your sperm count seems to be okay in the first semen analysis." b. "Only marijuana cigarettes affect sperm count." c. "Smoking can give you lung cancer, even though it has no effect on sperm." d. "Smoking can reduce the quality of your sperm."

ANS: D Use of tobacco, alcohol, and marijuana may affect sperm counts. "Your sperm count seems to be okay in the first semen analysis" is inaccurate. Sperm counts vary from day to day and depend on emotional and physical status and sexual activity. A single analysis may be inconclusive. A minimum of two analyses must be performed several weeks apart to assess male fertility.

1 Maternal serum screening is the diagnostic tool used for detecting chromosomal abnormalities in the fetus. It helps detect whether the fetus has an increased risk for Down syndrome or neural tube defects. Deviation in the number of chromosomes and a condition with an extra chromosome is considered a trisomy. Trisomy is an aneuploidic condition in which the chromosome count is observed as 47 (XXX). The incidence of Down syndrome is high in a child having a trisomy. The chromosome count for a female child with triploid condition is 69 (3N). The chromosome count for a female child with tetrasomy is 48 (XXXX). The chromosome count for a female child with tetraploidy condition is 92 (4N). The incidence of Down syndrome is not high in these conditions.

After reviewing the maternal serum screening results of a pregnant patient, the nurse infers that the patient may be carrying a child with Down syndrome. What is the potential finding in the report? 1 Trisomy 2 Triploid 3 Tetralogy 4 Tetraploidy

During the first trimester, a woman can expect which of the following changes in her sexual desire? A) An increase, because of enlarging breasts B) A decrease, because of nausea and fatigue C) No change D) An increase, because of increased levels of female hormones

B) A decrease, because of nausea and fatigue

The nurse caring for a newly pregnant woman would advise her that ideally prenatal care should begin: A) Before the first missed menstrual period. B) After the first missed menstrual period. C) After the second missed menstrual period. D) After the third missed menstrual period.

B) After the first missed menstrual period.

A woman who is 32 weeks pregnant is informed by the nurse that a danger sign of pregnancy could be: A) Constipation. B) Alteration in the pattern of fetal movement. C) Heart palpitations. D) Edema in the ankles and feet at the end of the day.

B) Alteration in the pattern of fetal movement.

To prevent gastrointestinal upset, clients should be instructed to take iron supplements: A) On a full stomach B) At bedtime C) After eating a meal D) With milk

B) At bedtime

Three servings of milk, yogurt, or cheese plus two servings of meat, poultry, or fish adequately supply the recommended amount of protein for a pregnant woman. Many patients are concerned about the increased levels of mercury in fish and may be afraid to include this source of nutrients in their diet. Sound advice by the nurse to assist the client in determining which fish is safe to consume would include: A) Canned white tuna is a preferred choice. B) Avoid shark, swordfish, and mackerel. C) Fish caught in local waterways are the safest. D) Salmon and shrimp contain high levels of mercury.

B) Avoid shark, swordfish, and mackerel.

Care management of a woman diagnosed with acute pelvic inflammatory disease (PID) most likely would include: A) Oral antiviral therapy. B) Bed rest in a semi-Fowler position. C) Antibiotic regimen continued until symptoms subside. D) Frequent pelvic examination to monitor the progress of healing.

B) Bed rest in a semi-Fowler position.

With regard to nutritional needs during lactation, a maternity nurse should be aware that: A) The mothers intake of vitamin C, zinc, and protein now can be lower than during pregnancy. B) Caffeine consumed by the mother accumulates in the infant, who may be unusually active and wakeful. C) Critical iron and folic acid levels must be maintained. D) Lactating women can go back to their prepregnant calorie intake.

B) Caffeine consumed by the mother accumulates in the infant, who may be unusually active and wakeful.

In teaching the pregnant adolescent about nutrition, the nurse should: A) Emphasize the need to eliminate common teen snack foods because they are too high in fat and sodium. B) Determine the weight gain needed to meet adolescent growth and add 35 lb. C) Suggest that she not eat at fast-food restaurants to avoid foods of poor nutritional value. D) Realize that most adolescents are unwilling to make dietary changes during pregnancy.

B) Determine the weight gain needed to meet adolescent growth and add 35 lb.

The nurse should have knowledge of the purpose of the pinch test. It is used to: A) Check the sensitivity of the nipples. B) Determine whether the nipple is everted or inverted. C) Calculate the adipose buildup in the abdomen. D) See whether the fetus has become inactive.

B) Determine whether the nipple is everted or inverted.

A pregnant woman at 18 weeks of gestation calls the clinic to report that she has been experiencing occasional backaches of mild-to-moderate intensity. The nurse would recommend that she: A) Do Kegel exercises B) Do pelvic rock exercises C) Use a softer mattress D) Stay in bed for 24 hours

B) Do pelvic rock exercises

During her gynecologic checkup, a 17-year-old girl states that recently she has been experiencing cramping and pain during her menstrual periods. The nurse would document this complaint as: A) Amenorrhea. B) Dysmenorrhea. C) Dyspareunia. D) Premenstrual syndrome (PMS).

B) Dysmenorrhea.

A pregnant woman experiencing nausea and vomiting should: A) Drink a glass of water with a fat-free carbohydrate before getting out of bed in the morning. B) Eat small, frequent meals (every 2 to 3 hours). C) Increase her intake of high-fat foods to keep the stomach full and coated. D) Limit fluid intake throughout the day.

B) Eat small, frequent meals (every 2 to 3 hours).

To detect human immunodeficiency virus (HIV), most laboratory tests focus on the: A) virus. B) HIV antibodies. C) CD4 counts. D) CD8 counts.

B) HIV antibodies.

A 25-year-old single woman comes to the gynecologists office for a follow-up visit related to her abnormal Papanicolaou (Pap) smear. The test revealed that the patient has human papillomavirus (HPV). The client asks, What is that? Can you get rid of it? Your best response is: A) Its just a little lump on your cervix. We can freeze it off. B) HPV stands for human papillomavirus. It is a sexually transmitted infection (STI) that may lead to cervical cancer. C) HPV is a type of early human immunodeficiency virus (HIV). You will die from this. D) You probably caught this from your current boyfriend. He should get tested for this.

B) HPV stands for human papillomavirus. It is a sexually transmitted infection (STI) that may lead to cervical cancer.

Which of the following statements about the various forms of hepatitis is accurate? A) A vaccine exists for hepatitis C but not for hepatitis B. B) Hepatitis A is acquired by eating contaminated food or drinking polluted water. C) Hepatitis B is less contagious than human immunodeficiency virus (HIV). D) The incidence of hepatitis C is decreasing.

B) Hepatitis A is acquired by eating contaminated food or drinking polluted water.

The viral sexually transmitted infection (STI) that affects most people in the United States today is: A) Herpes simplex virus type 2 (HSV-2). B) Human papillomavirus (HPV). C) Human immunodeficiency virus (HIV). D) Cytomegalovirus (CMV).

B) Human papillomavirus (HPV).

The most important reason for evaluating the pattern of weight gain in pregnancy is to: A) Prevent excessive adipose tissue deposits B) Identify potential nutritional problems or complications of pregnancy C) Assess the need to limit caloric intake in obese women D) Determine cultural influences on the woman's diet

B) Identify potential nutritional problems or complications of pregnancy

Which statement made by a lactating woman would lead the nurse to believe that the woman might have lactose intolerance? A) I always have heartburn after I drink milk. B) If I drink more than a cup of milk, I usually have abdominal cramps and bloating. C) Drinking milk usually makes me break out in hives. D) Sometimes I notice that I have bad breath after I drink a cup of milk.

B) If I drink more than a cup of milk, I usually have abdominal cramps and bloating.

Women with an inadequate weight gain during pregnancy are at higher risk of giving birth to an infant with: A) Spina bifida B) Intrauterine growth restriction C) Diabetes mellitus D) Down syndrome

B) Intrauterine growth restriction

Maternal nutritional status is an especially significant factor of the many factors that influence the outcome of pregnancy because: A) It is very difficult to adjust because of peoples' ingrained eating habits B) It is an important preventative measure for a variety of problems C) Women love obsessing about their weight and diets D) A woman's preconception weight becomes irrelevant

B) It is an important preventative measure for a variety of problems

What type of cultural concern is the most likely deterrent to many women seeking prenatal care? A) Religion B) Modesty C) Ignorance D) Belief that physicians are evil

B) Modesty

In understanding and guiding a woman through her acceptance of pregnancy, a maternity nurse should be aware that: A) Nonacceptance of the pregnancy very often equates to rejection of the child. B) Mood swings most likely are the result of worries about finances and a changed lifestyle as well as profound hormonal changes. C) Ambivalent feelings during pregnancy usually are seen only in emotionally immature or very young mothers. D) Conflicts such as not wanting to be pregnant or childrearing and career-related decisions need not be addressed during pregnancy because they will resolve themselves naturally after birth.

B) Mood swings most likely are the result of worries about finances and a changed lifestyle as well as profound hormonal changes.

While interviewing a 31-year-old woman before her routine gynecologic examination, the nurse collects data about the clients recent menstrual cycles. The nurse should collect additional information with which statement? A) The woman says her menstrual flow lasts 5 to 6 days. B) She describes her flow as very heavy. C) She reports that she has had a small amount of spotting midway between her periods for the past 2 months. D) She says the length of her menstrual cycle varies from 26 to 29 days.

B) She describes her flow as very heavy.

With regard to the initial physical examination of a woman beginning prenatal care, maternity nurses should be cognizant of: A) Only women who show physical signs or meet the sociologic profile should be assessed for physical abuse. B) The woman should empty her bladder before the pelvic examination is performed. C) The distribution, amount, and quality of body hair are of no particular importance. D) The size of the uterus is discounted in the initial examination.

B) The woman should empty her bladder before the pelvic examination is performed.

The U.S. Centers for Disease Control and Prevention (CDC) recommends that HPV be treated with client- applied: A) Miconazole ointment. B) Topical podofilox 0.5% solution or gel. C) Penicillin given intramuscularly for two doses. D) Metronidazole by mouth.

B) Topical podofilox 0.5% solution or gel.

Which statement about multifetal pregnancy is inaccurate? A) The expectant mother often develops anemia because the fetuses have a greater demand for iron. B) Twin pregnancies come to term with the same frequency as single pregnancies. C) The mother should be counseled to increase her nutritional intake and gain more weight. D) Backache and varicose veins often are more pronounced.

B) Twin pregnancies come to term with the same frequency as single pregnancies.

A 3-year-old girls mother is 6 months pregnant. What concern is this child likely to verbalize? A) How the baby will get out B) What the baby will eat C) Whether her mother will die D) What color eyes the baby has

B) What the baby will eat

To provide the patient with accurate information about dental care during pregnancy, maternity nurses should be aware that: A) Dental care can be dropped from the priority list because the woman has enough to worry about and is getting a lot of calcium anyway. B) Dental surgery, in particular, is contraindicated because of the psychologic stress it engenders. C) If dental treatment is necessary, the woman will be most comfortable with it in the second trimester. D) Dental care interferes with the expectant mothers need to practice conscious relaxation.

C) If dental treatment is necessary, the woman will be most comfortable with it in the second trimester.

2 Ultrasound testing is performed using high-frequency sound waves to obtain an image of the fetus. This technique is painless and a needle is not inserted in the womb. If the patient asks about the insertion of a needle in ultrasound, it implies that the patient did not understand about the technique and requires further teaching. A needle is used for an amniocentesis. These sound waves do not harm the fetus, and their frequency can be adjusted. This technique helps to identify the fetal abnormalities, because it shows the image of the fetus.

Before conducting the ultrasound scan, the nurse instructs the pregnant patient on the procedure. Which statement made by the patient indicates the need for further teaching? 1 "The sound waves are not harmful to the fetus at all." 2 "A needle is inserted in the abdomen during the scan." 3 "The frequency of the sound produced can be adjusted." 4 "Fetal anomalies can be identified by ultrasound scan."

A 27-year-old pregnant woman had a preconceptual body mass index (BMI) of 18.0. The nurse knows that this woman's total recommended weight gain during pregnancy should be at least: A) 20 kg (44 lb) B) 16 kg (35 lb) C) 12.5 kg (27.5 lb) D) 10 kg (22 lb)

C) 12.5 kg (27.5 lb)

If a patients normal prepregnancy diet contains 45 g of protein daily, how many more grams of protein should she consume per day during pregnancy? A) 5 B) 10 C) 25 D) 30

C) 25

The nurse should know that once human immunodeficiency virus (HIV) enters the body, seroconversion to HIV positivity usually occurs within: A) 6 to 10 days B) 2 to 4 weeks C) 6 to 8 weeks D) 6 months

C) 6 to 8 weeks

Which patient is most at risk for fibroadenoma of the breast? A) A 38-year-old woman B) A 50-year-old woman C) A 16-year-old girl D) A 27-year-old woman

C) A 16-year-old girl

Which meal would provide the most absorbable iron? A) Toasted cheese sandwich, celery sticks, tomato slices, and a grape drink B) Oatmeal, whole wheat toast, jelly, and low-fat milk C) Black bean soup, wheat crackers, orange sections, and prunes D) Red beans and rice, cornbread, mixed greens, and decaffeinated tea

C) Black bean soup, wheat crackers, orange sections, and prunes

The drug of choice for treatment of gonorrhea is: A) Penicillin G. B) Tetracycline. C) Ceftriaxone. D) Acyclovir.

C) Ceftriaxone.

When evaluating a patient for sexually transmitted infections (STIs), the nurse should be aware that the most common bacterial STI is: A) Gonorrhea. B) Syphilis. C) Chlamydia. D) Candidiasis.

C) Chlamydia.

With regard to follow-up visits for women receiving prenatal care, nurses should be aware that: A) The interview portions become more intensive as the visits become more frequent over the course of the pregnancy. B) Monthly visits are scheduled for the first trimester, every 2 weeks for the second trimester, and weekly for the third trimester. C) During the abdominal examination, the nurse should be alert for supine hypotension. D) For pregnant women, a systolic blood pressure (BP) of 130 and a diastolic BP of 80 is sufficient to be considered hypertensive.

C) During the abdominal examination, the nurse should be alert for supine hypotension.

In response to requests by the U.S. Public Health Service for new models of prenatal care, an innovative new approach to prenatal care known as centering pregnancy was developed. Which statement would accurately apply to the centering model of care? A) Group sessions begin with the first prenatal visit. B) At each visit, blood pressure, weight, and urine dipsticks are obtained by the nurse. C) Eight to 12 women are placed in gestational-age cohort groups. D) Outcomes are similar to those of traditional prenatal care.

C) Eight to 12 women are placed in gestational-age cohort groups.

Which diagnostic test is used to confirm a suspected diagnosis of breast cancer? A) Mammogram B) Ultrasound C) Fine-needle aspiration (FNA) D) CA 15.3

C) Fine-needle aspiration (FNA)

With regard to endometriosis, nurses should be aware that: A) It is characterized by the presence and growth of endometrial tissue inside the uterus. B) It is found more often in African-American women than in white or Asian women. C) It may worsen with repeated cycles or remain asymptomatic and disappear after menopause. D) It is unlikely to affect sexual intercourse or fertility.

C) It may worsen with repeated cycles or remain asymptomatic and disappear after menopause.

When assessing a patient for amenorrhea, the nurse should be aware that this is unlikely to be caused by: A) Anatomic abnormalities B) Type 1 diabetes mellitus. C) Lack of exercise. D) Hysterectomy.

C) Lack of exercise.

Pregnant adolescents are at high risk for _____ because of lower body mass indices (BMIs) and fad dieting. A) Obesity B) Diabetes C) Low-birth-weight babies D) High-birth-weight babies

C) Low-birth-weight babies

The nurse providing education regarding breast care should explain to the woman that fibrocystic changes in breasts are: A) A disease of the milk ducts and glands in the breasts. B) A premalignant disorder characterized by lumps found in the breast tissue. C) Lumpiness with pain and tenderness found in varying degrees in the breast tissue of healthy women during menstrual cycles. D) Lumpiness accompanied by tenderness after menses.

C) Lumpiness with pain and tenderness found in varying degrees in the breast tissue of healthy women during menstrual cycles.

A 36-year-old woman has been given a diagnosis of uterine fibroids. When planning care for this patient, the nurse should know that: A) Fibroids are malignant tumors of the uterus that require radiation or chemotherapy. B) Fibroids increase in size during the perimenopausal period. C) Menorrhagia is a common finding. D) The woman is unlikely to become pregnant as long as the fibroids are in her uterus.

C) Menorrhagia is a common finding.

Management of primary dysmenorrhea often requires a multifaceted approach. The nurse who provides care for a client with this condition should be aware that the optimal pharmacologic therapy for pain relief is: A) Acetaminophen. B) Oral contraceptives (OCPs). C) Nonsteroidal antiinflammatory drugs (NSAIDs). D) Aspirin.

C) Nonsteroidal antiinflammatory drugs (NSAIDs).

With regard to the initial visit with a client who is beginning prenatal care, nurses should be aware that: A) The first interview is a relaxed, get-acquainted affair in which nurses gather some general impressions. B) If nurses observe handicapping conditions, they should be sensitive and not inquire about them because the client will do that in her own time. C) Nurses should be alert to the appearance of potential parenting problems, such as depression or lack of family support. D) Because of legal complications, nurses should not ask about illegal drug use; that is left to physicians.

C) Nurses should be alert to the appearance of potential parenting problems, such as depression or lack of family support.

While taking a diet history, the nurse might be told that the expectant mother has cravings for ice chips, cornstarch, and baking soda. This represents a nutritional problem known as: A) Preeclampsia B) Pyrosis C) Pica D) Purging

C) Pica

When evaluating a patient whose primary complaint is amenorrhea, the nurse must be aware that lack of menstruation is most often the result of: A) Stress B) Excessive exercise C) Pregnancy D) Eating disorders.

C) Pregnancy

Which statement concerning cyclic perimenstrual pain and discomfort (CPPD) is accurate? A) Premenstrual dysphoric disorder (PMDD) is a milder form of premenstrual syndrome (PMS) and more common in younger women. B) Secondary dysmenorrhea is more intense and medically significant than primary dysmenorrhea. C) Premenstrual syndrome is a complex, poorly understood condition that may include any of a hundred symptoms. D) The causes of PMS have been well established.

C) Premenstrual syndrome is a complex, poorly understood condition that may include any of a hundred symptoms.

The nurse should be aware that the partners main role in pregnancy is to: A) Provide financial support. B) Protect the pregnant woman from old wives tales. C) Support and nurture the pregnant woman. D) Make sure the pregnant woman keeps prenatal appointments.

C) Support and nurture the pregnant woman.

The labor and delivery nurse is preparing a bariatric patient for an elective cesarean birth. Which piece of specialized equipment is unnecessary when providing care for this pregnant woman. A) Extra long surgical instruments B) Wide surgical table C) Temporal thermometer D) Increased diameter blood pressure cuff

C) Temporal thermometer

A pregnant woman's diet consists almost entirely of whole grain breads and cereals, fruits, and vegetables. The nurse would be most concerned about this woman's intake of: A) Calcium B) Protein C) Vitamin B12 D) Folic acid

C) Vitamin B12

After a mastectomy a woman should be instructed to perform all of the following except: A) Emptying surgical drains twice a day and as needed. B) Avoiding lifting more than 4.5 kg (10 lb) or reaching above her head until given permission by her surgeon. C) Wearing clothing with snug sleeves to support the tissue of the arm on the operative side. D) Reporting immediately if inflammation develops at the incision site or in the affected arm.

C) Wearing clothing with snug sleeves to support the tissue of the arm on the operative side.

A nurse practitioner performs a clinical breast examination on a woman diagnosed with fibroadenoma. The nurse knows that fibroadenoma is characterized by: A) Inflammation of the milk ducts and glands behind the nipples. B) Thick, sticky discharge from the nipple of the affected breast. C) Lumpiness in both breasts that develops 1 week before menstruation. D) A single lump in one breast that can be expected to shrink as the woman ages.

D) A single lump in one breast that can be expected to shrink as the woman ages.

With regard to a womans reordering of personal relationships during pregnancy, the maternity nurse should understand that: A) Because of the special motherhood bond, a woman's relationship with her mother is even more important than with the father of the child. B) Nurses need not get involved in any sexual issues the couple has during pregnancy, particularly if they have trouble communicating them to each other. C) Women usually express two major relationship needs during pregnancy: feeling loved and valued and having the child accepted by the father. D) The woman's sexual desire is likely to be highest in the first trimester because of the excitement and because intercourse is physically easier.

C) Women usually express two major relationship needs during pregnancy: feeling loved and valued and having the child accepted by the father.=

A pregnant woman at 10 weeks of gestation jogs three or four times per week. She is concerned about the effect of exercise on the fetus. The nurse should inform her: A) You don't need to modify your exercising any time during your pregnancy. B) Stop exercising because it will harm the fetus. C) You may find that you need to modify your exercise to walking later in your pregnancy, around the seventh month. D) Jogging is too hard on your joints; switch to walking now.

C) You may find that you need to modify your exercise to walking later in your pregnancy, around the seventh month.

A woman who is 14 weeks pregnant tells the nurse that she always had a glass of wine with dinner before she became pregnant. She has abstained during her first trimester and would like to know if it is safe for her to have a drink with dinner now. The nurse would tell her: A) Since you're in your second trimester, there's no problem with having one drink with dinner. B) One drink every night is too much. One drink three times a week should be fine. C) Since you're in your second trimester, you can drink as much as you like. D) Because no one knows how much or how little alcohol it takes to cause fetal problems, the best course is to abstain throughout your pregnancy.

D) Because no one knows how much or how little alcohol it takes to cause fetal problems, the best course is to abstain throughout your pregnancy.

Nafarelin is currently used as a treatment for mild-to-severe endometriosis. The nurse should tell a woman taking this medication that the drug: A) Stimulates the secretion of gonadotropin-releasing hormone (GnRH), thereby stimulating ovarian activity. B) Should be sprayed into one nostril every other day. C) Should be injected into subcutaneous tissue BID. D) Can cause her to experience some hot flashes and vaginal dryness.

D) Can cause her to experience some hot flashes and vaginal dryness.

A pregnant womans diet history indicates that she likes the following list of foods. The nurse would encourage this woman to consume more of which food to increase her calcium intake? A) Fresh apricots B) Canned clams C) Spaghetti with meat sauce D) Canned sardines

D) Canned sardines

The nurse providing care in a womens health care setting must be aware regarding which sexually transmitted infection that can be successfully treated and cured? A) Herpes B) Acquired immunodeficiency syndrome (AIDS) C) Venereal warts D) Chlamydia

D) Chlamydia

The major source of nutrients in the diet of a pregnant woman should be composed of: A) Simple sugars B) Fats C) Fiber D) Complex carbohydrates

D) Complex carbohydrates

When counseling a client about getting enough iron in her diet, the maternity nurse should tell her that: A) Milk, coffee, and tea aid iron absorption if consumed at the same time as iron. B) Iron absorption is inhibited by a diet rich in vitamin C. C) Iron supplements are permissible for children in small doses. D) Constipation is common with iron supplements.

D) Constipation is common with iron supplements.

The nurse must watch for what common complications in a patient who has undergone a transverse rectus abdominis myocutaneous (TRAM) flap? A) Axillary edema and tissue necrosis B) Delayed wound healing and muscle contractions C) Delayed wound healing and axillary edema D) Delayed wound healing and hematoma

D) Delayed wound healing and hematoma

A woman in week 34 of pregnancy reports that she is very uncomfortable because of heartburn. The nurse would suggest that the woman: A) Substitute other calcium sources for milk in her diet. B) Lie down after each meal. C) Reduce the amount of fiber she consumes. D) Eat five small meals daily.

D) Eat five small meals daily.

Nutrition is one of the most significant factors influencing the outcome of a pregnancy. It is an alterable and important preventive measure for various potential problems, such as low birth weight and prematurity. While completing the physical assessment of the pregnant client, the nurse can evaluate the clients nutritional status by observing a number of physical signs. Which sign would indicate that the client has unmet nutritional needs? A) Normal heart rate, rhythm, and blood pressure B) Bright, clear, shiny eyes C) Alert, responsive, and good endurance D) Edema, tender calves, and tingling

D) Edema, tender calves, and tingling

The phenomenon of someone other than the mother-to-be experiencing pregnancy-like symptoms such as nausea and weight gain applies to the: A) Mother of the pregnant woman B) Couple's teenage daughter C) Sister of the pregnant woman D) Expectant father

D) Expectant father

A pregnant woman's diet may not meet her need for folates. A good source of this nutrient is: A) Chicken B) Cheese C) Potatoes D) Green leafy vegetables

D) Green leafy vegetables

While you are assessing the vital signs of a pregnant woman in her third trimester, the patient complains of feeling faint, dizzy, and agitated. Which nursing intervention is appropriate? A) Have the patient stand up and retake her blood pressure. B) Have the patient sit down and hold her arm in a dependent position. C) Have the patient lie supine for 5 minutes and recheck her blood pressure on both arms. D) Have the patient turn to her left side and recheck her blood pressure in 5 minutes.

D) Have the patient turn to her left side and recheck her blood pressure in 5 minutes.

A woman is 3 months pregnant. At her prenatal visit, she tells the nurse that she doesn't know what is happening; one minute shes happy that she is pregnant, and the next minute she cries for no reason. Which response by the nurse is most appropriate? A) Don't worry about it; you'll feel better in a month or so. B) Have you talked to your husband about how you feel? C) Perhaps you really don't want to be pregnant. D) Hormonal changes during pregnancy commonly result in mood swings.

D) Hormonal changes during pregnancy commonly result in mood swings.

For what reason would breastfeeding be contraindicated? A) Hepatitis B B) Everted nipples C) History of breast cancer 3 years ago D) Human immunodeficiency virus (HIV) positive

D) Human immunodeficiency virus (HIV) positive

What represents a typical progression through the phases of a woman's establishing a relationship with the fetus? A) Accepts the fetus as distinct from herself accepts the biologic fact of pregnancy has a feeling of caring and responsibility B) Fantasizes about the child's gender and personality views the child as part of herself becomes introspective C) Views the child as part of herself has feelings of well-being accepts the biologic fact of pregnancy D) I am pregnant.I am going to have a baby.I am going to be a mother.

D) I am pregnant.I am going to have a baby.I am going to be a mother.

To determine the cultural influence on a patients diet, the nurse should first: A) Evaluate the patients weight gain during pregnancy B) Assess the socioeconomic status of the patient C) Discuss the four food groups with the patient D) Identify the food preferences and methods of food preparation common to that culture

D) Identify the food preferences and methods of food preparation common to that culture

A woman has come to the clinic for preconception counseling because she wants to start trying to get pregnant in 3 months. She can expect the following advice: A) Discontinue all contraception now B) Lose weight so that you can gain more during pregnancy C) You may take any medications you have been taking regularly D) Make sure that you include adequate folic acid in your diet

D) Make sure that you include adequate folic acid in your diet

One of the alterations in cyclic bleeding that occurs between periods is called: A) Oligomenorrhea. B) Menorrhea. C) Leiomyoma. D) Metrorrhagia.

D) Metrorrhagia.

With regard to dysmenorrhea, nurses should be aware that: A) It is more common in older women. B) It is more common in leaner women who exercise strenuously. C) Symptoms can begin at any point in the ovulatory cycle. D) Pain usually occurs in the suprapubic area or lower abdomen.

D) Pain usually occurs in the suprapubic area or lower abdomen.

The two primary areas of risk for sexually transmitted infections (STIs) are: A) Sexual orientation and socioeconomic status. B) Age and educational level. C) Large number of sexual partners and race. D) Risky sexual behaviors and inadequate preventive health behaviors.

D) Risky sexual behaviors and inadequate preventive health behaviors.

The nurse who is teaching a group of women about breast cancer would tell the women that: A) Risk factors identify more than 50% of women who will develop breast cancer. B) Nearly 90% of lumps found by women are malignant. C) One in 10 women in the United States will develop breast cancer in her lifetime. D) The exact cause of breast cancer is unknown.

D) The exact cause of breast cancer is unknown.

A 22-year-old woman pregnant with a single fetus has a preconception body mass index (BMI) of 24. When she was seen in the clinic at 14 weeks of gestation, she had gained 1.8 kg (4 lb) since conception. How would the nurse interpret this? A) This weight gain indicates possible gestational hypertension. B) This weight gain indicates that the woman's infant is at risk for intrauterine growth restriction (IUGR). C) This weight gain cannot be evaluated until the woman has been observed for several more weeks. D) The woman's weight gain is appropriate for this stage of pregnancy.

D) The woman's weight gain is appropriate for this stage of pregnancy.

In her work with pregnant women of various cultures, a nurse practitioner has observed various practices that seemed strange or unusual. She has learned that cultural rituals and practices during pregnancy seem to have one purpose in common. Which statement best describes that purpose? A) To promote family unity B) To ward off the evil eye C) To appease the gods of fertility D) To protect the mother and fetus during pregnancy

D) To protect the mother and fetus during pregnancy

When a nurse is counseling a woman for primary dysmenorrhea, which nonpharmacologic intervention might be recommended? A) Increasing the intake of red meat and simple carbohydrates B) Reducing the intake of diuretic foods such as peaches and asparagus C) Temporarily substituting physical activity for a sedentary lifestyle D) Using a heating pad on the abdomen to relieve cramping

D) Using a heating pad on the abdomen to relieve cramping

Which symptom is considered a first-trimester warning sign and should be reported immediately by the pregnant woman to her health care provider? A) Nausea with occasional vomiting B) Fatigue C) Urinary frequency D) Vaginal bleeding

D) Vaginal bleeding

Which vitamins or minerals can lead to congenital malformations of the fetus if taken in excess by the mother? A) Zinc B) Vitamin D C) Folic Acid D) Vitamin A

D) Vitamin A

When discussing work and travel during pregnancy with a pregnant patient, nurses should instruct them that: A) Women should sit for as long as possible and cross their legs at the knees from time to time for exercise. B) Women should avoid seat belts and shoulder restraints in the car because they press on the fetus. C) Metal detectors at airport security checkpoints can harm the fetus if the woman passes through them a number of times. D) While working or traveling in a car or on a plane, women should arrange to walk around at least every hour or so.

D) While working or traveling in a car or on a plane, women should arrange to walk around at least every hour or so.

Which pregnant woman should restrict her weight gain during pregnancy? A) Woman pregnant with twins B) Woman in early adolescence C) Woman shorter than 62 inches or 157 cm D) Woman who was 20 pounds overweight before pregnancy

D) Woman who was 20 pounds overweight before pregnancy

A healthy 60-year-old African-American woman regularly receives her health care at the clinic in her neighborhood. She is due for a mammogram. At her previous clinic visit, her physician, concerned about the 3- week wait at the neighborhood clinic, made an appointment for her to have a mammogram at a teaching hospital across town. She did not keep her appointment and returned to the clinic today to have the nurse check her blood pressure. What would be the most appropriate statement for the nurse to make to this patient? A) Do you have transportation to the teaching hospital so that you can get your mammogram? B) I'm concerned that you missed your appointment; let me make another one for you. C) Its very dangerous to skip your mammograms; your breasts need to be checked. D) Would you like me to make an appointment for you to have your mammogram here?

D) Would you like me to make an appointment for you to have your mammogram here?

The recommended treatment for the prevention of human immunodeficiency virus (HIV) transmission to the fetus during pregnancy is: A) Acyclovir B) Ofloxacin C) Podophyllin D) Zidovudine

D) Zidovudine

1 Color blindness is an X-linked recessive inheritance. The genes responsible for the disease are carried on the X chromosome. In this scenario, the patient does not have the genes that cause color blindness. However, the patient's partner's mother has color blindness. Because color blindness is an autosomal recessive disorder, this implies that the patient's partner is affected and is color-blind. The patient's son acquires the unaffected X chromosome from the mother and Y chromosome from the father. Therefore this patient's son will not be affected and will not develop color blindness. Males cannot be the carriers for X-linked chromosomal disorders, but they are always expressed in them. A 25% and 50% chance of developing color blindness in the patient's son occurs if the mother is a carrier for the disease.

During a genetic counseling session, a patient discloses that her partner's mother has color blindness. The laboratory results show that the patient does not have the genes that are responsible for color blindness. What does the nurse infer from this? The patient's son: 1 Will not demonstrate any color blindness. 2 Will become a carrier for color blindness. 3 Has a 25% chance of having color blindness. 4 Has a 50% chance of having color blindness.

3 The fetal heart starts beating by the end of the fifth week. Two arteries carry blood from the embryo to the chorionic villi, and one vein returns the blood to the embryo. Deoxygenated blood leaves the fetus through the umbilical arteries and enters the placenta, where it is oxygenated. Oxygenated blood leaves the placenta through the umbilical vein, which enters the fetus via the umbilical cord. It is rare for there to be one vein and one artery. Approximately 1% of umbilical cords contain only two vessels: one artery and one vein. This occurrence is sometimes associated with congenital malformations. Therefore the presence of two veins and one artery and two veins and two arteries is inappropriate.

How many veins and arteries are present between the maternal and the fetal circulatory system by the fifth week of pregnancy? 1 One vein and one artery 2 Two veins and one artery 3 One vein and two arteries 4 Two veins and two arteries

10, 9, 40, 280 Pregnancy lasts approximately 10 lunar months, 9 calendar months, 40 weeks, 280 days. Length of pregnancy is computed from the first day of the last menstrual period (LMP) until the day of birth.

List the time span in lunar months, calendar months, weeks, and days that indicates the appropriate length for a normal pregnancy. Record your response as whole numbers separated by commas (ex. 2, 4, 6, 8). _______________

1 Pregnant women often experience hypotension when they lie on their back (in the supine position). The blood that is trying to return to the right atrium is diminished because the uterus is compressing the vena cava. Therefore the nurse should have the patient to lie on either side to rest. This will reduce the uterine pressure on the right atrium and helps in optimal circulation. Aerobic exercise will not be helpful in reducing the pressure caused by uterine compression. Making the patient stand up and take deep breaths will worsen the symptoms of hypotension. Making the patient lie in the supine position will further increase the uterine compression and hypotension.

The blood pressure of a pregnant patient becomes low when the patient lies on the back. What would be the best nursing intervention to maintain normal blood pressure in the patient? 1 Position the patient to lie on the left side and rest. 2 Suggest that the patient perform aerobic exercises daily. 3 Have the patient stand up and take a deep breath. 4 Tell the patient to lie straight facing up and take frequent rest periods.

2 Genetic testing of the couple may determine whether the parents of the newborn are carriers for the mutation. Phenylketonuria can be seen in the newborn only if both parents are carriers for the disorder. Fetal sonography is a technique of imaging the fetus in the uterus using high-frequency sound waves. Chorionic villus sampling is a prenatal diagnostic technique to determine chromosomal disorders in the fetus. Maternal serum sampling is carried out on the mother through a blood test to screen the high risk of carrying a fetus with neural tube defects or genetic abnormalities.

The genetic screening of a newborn is indicative of phenylketonuria. The nurse learns that the couple has no history of genetic disorders in the past three generations. What other tests does the nurse suggest to the couple to determine the reason for the genetic disorder? 1 Sonography of the fetus 2 Genetic test of the couple 3 Chorionic villus sampling 4 Maternal serum screening

1 The risk for Down syndrome increases in the fetus as the age of the pregnant mother gets older. Because the patient is older than 35 years of age, the fetus is at an increased risk for having Down syndrome. The development of sickle cell anemia is not related to the age of the mother. It is caused by inheriting the defective genes from the parents. Neural tube defect in the fetus results from insufficient intake of folic acid during pregnancy. HD is an autosomal dominant disorder caused by a mutation on the normal gene and is not related to the age of the patient.

The nurse is assessing a 40-year-old patient who is 12 weeks pregnant. What should the nurse anticipate based on the patient's age? 1 The fetus is at a risk for Down syndrome. 2 The patient is at a risk for sickle cell anemia. 3 The fetus is at a risk for neural tube defects. 4 The patient is at a risk for Huntington's disease (HD).

1 Patients with uncontrolled diabetes are at a higher risk for complications associated with pregnancy. If the pregnant mother develops uncontrolled hyperglycemia, this may produce hyperglycemia in the fetus. This in turn stimulates fetal hyperinsulinemia and islet cell hyperplasia. Hyperinsulinemia prevents fetal lung maturation and places the neonate at an increased risk for respiratory distress. African-American patients have an increased chance of having dizygotic twins. They are not associated with an increased risk for pregnancy-related complications. Patients within the age-group of 30 to 33 years are not at risk for complications associated with pregnancy. Alcohol consumption during pregnancy leads to respiratory complications and fetal alcohol syndrome. However, patients with a history of alcohol consumption do not usually have pregnancy-related complications.

The nurse is assessing a group of pregnant women at a community health center. Which patients would be at highest risk for pregnancy-related complications? The patient: 1 With uncontrolled diabetes mellitus. 2 Who is of African-American descent. 3 Who is between 30 and 33 years old. 4 With history of alcohol consumption.

4 Down syndrome is an intellectual disability caused by aneuploidy. Aneuploidy is a condition characterized by the presence or absence of an extra chromosome. Therefore the presence of extra chromosome 21 causes Down syndrome in the newborn. The deficiency of folic acid results in neural tube defects in the newborn. X-linked recessive gene disorder results from the presence of a defective gene on the X-chromosome of a carrier mother. Down syndrome is not caused by inheritance of an autosomal recessive gene. It occurs because of the defect in the chromosomal count in an individual.

The nurse is interacting with the parents of a newborn who is diagnosed with Down syndrome. What explanation should the nurse provide to the parents about the genetic disorder? Down syndrome is caused by: 1 A decreased intake of folic acid." 2 An X-linked recessive gene issue." 3 An autosomal recessive gene." 4 The presence of an extra chromosome."

4 Stickler syndrome is an autosomal dominant inheritance caused by the mutation on the normal gene of the individual. Because the syndrome shows few symptoms, the nurse should assess the family history of the patient. The nurse should also ask about whether the patient has other children and if they have any birth defects as a result of the syndrome. The age of the parent plays a vital role during the diagnosis of Down syndrome. Therefore it is not as important to know about the age of the patient's mother and father. Socioeconomic status is not a cause for genetic abnormalities, so the nurse should not include the question about the socioeconomic status of the patient during counseling.

The nurse is providing genetic counseling for a patient with Stickler syndrome. What questions should the nurse ask the patient? 1 "Was your father older than 50 years of age when you were born?" 2 "Was your mother older than 35 years of age when you were born?" 3 "May I ask what the socioeconomic status of your family is?" 4 "Have you ever had any other children with birth defects?"

2 Ultrasound scan or fetal ultrasound is the technique used for visualizing the fetus and the internal structures for prenatal analysis. In this technique, high-frequency sound waves produce the image of the fetus without harming the fetal internal organs. Therefore this technique is used to identify the presence of twins in the patient's womb. Human placental lactogen changes the metabolism of the mother and supplies energy to the fetus. Cytogenetic testing helps find the genetic abnormalities and that caused by changes in the chromosomes. Amniotic fluid protects the fetus from injuries. These tests do not help determine the presence of twins in the patient's womb.

The nurse is reviewing the diagnostic test results with a pregnant patient and informs the patient that she is going to have twins. Based on which diagnostic test did the nurse make such a conclusion? 1 Human placental lactogen 2 Ultrasound results 3 Cytogenetic testing 4 Amniotic fluid levels

3 Religion does not play a major role in the fetal development. Therefore it is not required to ask whether the patient follows Christianity. Before conducting prenatal testing, the nurse should assess the socioeconomic status of the patient's family. It helps to know the factors that may affect fetal growth and development. Therefore the nurse should ask questions to obtain the information regarding the socioeconomic status of the patient. The nurse should ask about the partner's age to assess the risk for genetic disorders such as Down syndrome. Asking whether the patient regularly drinks alcohol helps assess the risk for fetal alcohol spectrum disorder. Asking about the profession of the patient's partner provides the clue for the risk for carcinogenesis.

The nurse is teaching a student nurse about questions that can be asked of the patient before undergoing prenatal testing. The nurse then asked the student to write the set of questions on a piece of paper. Which question written by the student nurse indicates the need for further teaching? 1 "What is the age of your partner?" 2 "Do you smoke or drink regularly?" 3 "Do you believe in Christianity?" 4 "What is your partner's profession?

1 Warfarin (Coumadin) is the drug of choice given to children for the treatment of thromboembolic events. However, for treatment to be effective and to prevent adverse effects, the dosage of warfarin (Coumadin) may have to be customized for each patient based on his or her genotype. This drug does not have to be given on an empty stomach. Warfarin (Coumadin) is not given as a test dose. Some antibiotics may be administered by testing a small dose first to determine hypersensitivity, but not warfarin (Coumadin). This medication is not given by the intramuscular route. It is an anticoagulant, and giving the medication through this route may cause bleeding.

The primary health care provider instructs the nurse to administer warfarin (Coumadin) to a child with venous thrombosis. What precaution should the nurse take to prevent possible severe adverse drug reactions in the child? Administer: 1 The genotype-guided dose of the drug. 2 The drug orally on an empty stomach. 3 A test dosage before the medication. 4 The drug by the intramuscular route.

3 Two milestones that occur at 20 weeks are the appearance of the vernix caseosa and lanugo. The primary lung and urethral buds appear at 6 weeks of gestation. The vagina is open or the testes are in position for descent into the scrotum at 16 weeks. The appearance of smooth skin occurs at 28 weeks, and subcutaneous fat begins to collect at 30 to 31 weeks.

The student nurse is giving a presentation about milestones in embryonic development. Which information should the student include? 1 At 8 weeks of gestation, primary lung and urethral buds appear. 2 At 12 weeks of gestation, the vagina is open or the testes are in position for descent into the scrotum. 3 At 20 weeks of gestation, the vernix caseosa and lanugo appear. 4 At 24 weeks of gestation, the skin is smooth, and subcutaneous fat is beginning to collect.

C, E A female patient with monosomy of the X chromosome and missing paternal X or Y chromosome has a genetic makeup of 45,X, which indicates Turner syndrome. Therefore, the nurse expects that the patient will have associated webbing of the neck and lymphedema of the hands. Such patients are generally short in stature and have low-set ears with a low hairline in the back.

What findings does the nurse expect in a female patient whose genetic report reveals monosomy of the X chromosome with a missing paternal X or Y chromosome? Select all that apply. A Tall stature B High-set ears C Webbing of the neck D High hairline in the back E Lymphedema of the hands

2 The sperm remains viable in the female reproductive system for 3 to 5 days after sexual intercourse. Within this time a sperm fuses with an ova to form a zygote. The sperm is no longer viable after 5 days. Therefore they do not remain viable after 4 to 8, and 5 to 9 days after intercourse.

What is the duration for which the sperm remains viable in the female reproductive system? 1 2 to 4 days after intercourse 2 3 to 5 days after intercourse 3 4 to 8 days after intercourse 4 5 to 9 days after intercourse

1 The gestation period of a fetus is 266 days from the date of conception and 280 days from the last menstrual period. Calculating the gestational period after conception from June 20, the expected date of delivery is March 13, 2013. March 27, 2013, is the expected date of delivery if June 20, 2012, is the first day of the last menstrual period. February 13 and 27 are not the expected date of delivery unless the baby is a preterm infant.

What is the expected delivery date of a patient whose conception is reported on June 20, 2012? 1 March 13, 2013 2 March 27, 2013 3 February 13, 2013 4 February 27, 2013

2 Mosaicism is a condition in which the individual may have an extra chromosome in some of the cells. This disorder is usually associated with sex chromosome. It is detected to be present on an autosomal gene and results in Down syndrome in the majority of individuals. Neural tube defects are caused by the deficiency of folic acid during pregnancy. Congenital heart defect is a multifactorial inheritance caused by environmental and chromosomal defects. The child will not be born with normal intelligence.

Which findings can be observed in the individuals with mosaicism on an autosomal gene? 1 Neural tube defect 2 Down syndrome 3 Congenital heart defect 4 Normal intelligence

2 Aneuploidy is an abnormality of number that also is the leading genetic cause of cognitive impairment. Chromosomal abnormalities occur in less than 1% of newborns. Down syndrome is the most common form of trisomal abnormality, an abnormality of chromosome number (47 chromosomes). Unbalanced translocation is an abnormality of chromosome structure that often has serious clinical effects.

With regard to abnormalities of chromosomes, nurses should be aware that: 1 they occur in approximately 10% of newborns. 2 abnormalities of number are a major cause of pregnancy loss. 3 Down syndrome is a result of an abnormal chromosomal structure. 4 unbalanced translocation results in a mild abnormality that the child will outgrow.

Compared with contraction stress test (CST), nonstress test (NST) for antepartum fetal assessment: a. Has no known contraindications. b. Has fewer false-positive results. c. Is more sensitive in detecting fetal compromise. d. Is slightly more expensive.

a. Has no known contraindications.

Nurses should be aware that the biophysical profile (BPP): a. Is an accurate indicator of impending fetal death. b. Is a compilation of health risk factors of the mother during the later stages of pregnancy. c. Consists of a Doppler blood flow analysis and an amniotic fluid index. d. Involves an invasive form of ultrasound examination.

a. Is an accurate indicator of impending fetal death.

Transvaginal ultrasonography is often performed during the first trimester. While preparing your 6-week gestation patient for this procedure, she expresses concerns over the necessity for this test. The nurse should explain that this diagnostic test may be indicated for a number of situations (Select all that apply). a. Multifetal gestation b. Obesity c. Fetal abnormalities d. Amniotic fluid volume e. Ectopic pregnancy

a. Multifetal gestation b. Obestiy c. Fetal abnormalities e. Ectopic pregnancy

Which analysis of maternal serum may predict chromosomal abnormalities in the fetus? a. Multiple-marker screening b. Lecithin/sphingomyelin (L/S) ratio c. Biophysical profile d. Type and crossmatch of maternal and fetal serum

a. Multiple-marker screening

A woman is undergoing a nipple-stimulated contraction stress test (CST). She is having contractions that occur every 3 minutes. The fetal heart rate (FHR) has a baseline of approximately 120 beats/min without any decelerations. The interpretation of this test is said to be: a. Negative. b. Positive. c. Satisfactory. d. Unsatisfactory.

a. Negative.

Fetal congenital anomalies a. Polyhydramnios b. Intrauterine growth restriction (maternal cause) c. Oligohydramnios d. Chromosomal abnormalities e. Intrauterine growth restriction (fetoplacental cause)

a. Polyhydramnios

Intrauterine growth restriction (IUGR) is associated with numerous pregnancy-related risk factors(Select all that apply). a. Poor nutrition b. Maternal collagen disease c. Gestational hypertension d. Premature rupture of membranes e. Smoking

a. Poor nutrition b. Maternal collagen disease c. Gestational hypertension e. Smoking

A pregnant woman's biophysical profile score is 8. She asks the nurse to explain the results. The nurses best response is: a. The test results are within normal limits. b. Immediate delivery by cesarean birth is being considered. c. Further testing will be performed to determine the meaning of this score. d. An obstetric specialist will evaluate the results of this profile and, within the next week, will inform you of your options regarding delivery.

a. The test results are within normal limits.

A 39-year-old primigravida thinks that she is about 8 weeks pregnant, although she has had irregular menstrual periods all her life. She has a history of smoking approximately one pack of cigarettes a day, but she tells you that she is trying to cut down. Her laboratory data are within normal limits. What diagnostic technique could be used with this pregnant woman at this time? a. Ultrasound examination b. Maternal serum alpha-fetoprotein (MSAFP) screening c. Amniocentesis d. Nonstress test (NST)

a. Ultrasound examination

A client asks her nurse, My doctor told me that he is concerned with the grade of my placenta because I am overdue. What does that mean? The best response by the nurse is: a. Your placenta changes as your pregnancy progresses, and it is given a score that indicates the amount of calcium deposits it has. The more calcium deposits, the higher the grade, or number, that is assigned to the placenta. It also means that less blood and oxygen can be delivered to your baby. b. Your placenta isn't working properly, and your baby is in danger. c. This means that we will need to perform an amniocentesis to detect if you have any placental damage. d. Don't worry about it. Everything is fine.

a. Your placenta changes as your pregnancy progresses, and it is given a score that indicates the amount of calcium deposits it has. The more calcium deposits, the higher the grade, or number, that is assigned to the placenta. It also means that less blood and oxygen can be delivered to your baby.

A 41-week pregnant multigravida presents in the labor and delivery unit after a nonstress test indicated that her fetus could be experiencing some difficulties in utero. Which diagnostic tool would yield more detailed information about the fetus? a. Ultrasound for fetal anomalies b. Biophysical profile (BPP) c. Maternal serum alpha-fetoprotein (MSAFP) screening d. Percutaneous umbilical blood sampling (PUBS)

b. Biophysical profile (BPP)

A woman has been diagnosed with a high risk pregnancy. She and her husband come into the office in a very anxious state. She seems to be coping by withdrawing from the discussion, showing declining interest. The nurse can best help the couple by: a. Telling her that the physician will isolate the problem with more tests. b. Encouraging her and urging her to continue with childbirth classes. c. Becoming assertive and laying out the decisions the couple needs to make. d. Downplaying her risks by citing success rate studies.

b. Encouraging her and urging her to continue with childbirth classes.

Which nursing intervention is necessary before a second-trimester transabdominal ultrasound? a. Place the woman NPO for 12 hours. b. Instruct the woman to drink 1 to 2 quarts of water. c. Administer an enema. d. Perform an abdominal preparation.

b. Instruct the woman to drink 1 to 2 quarts of water.

Smoking, alcohol, illicit drug use a. Polyhydramnios b. Intrauterine growth restriction (maternal cause) c. Oligohydramnios d. Chromosomal abnormalities e. Intrauterine growth restriction (fetoplacental cause)

b. Intrauterine growth restriction (maternal cause)

In the first trimester, ultrasonography can be used to gain information on: a. Amniotic fluid volume. b. Location of Gestational sacs c. Placental location and maturity. d. Cervical length.

b. Location of Gestational sacs

Risk factors tend to be interrelated and cumulative in their effect. While planning the care for a laboring client with diabetes mellitus, the nurse is aware that she is at a greater risk for: a. Oligohydramnios. b. Polyhydramnios. c. Postterm pregnancy. d. Chromosomal abnormalities.

b. Polyhydramnios.

Nurses should be aware of the strengths and limitations of various biochemical assessments during pregnancy, including that: a. Chorionic villus sampling (CVS) is becoming more popular because it provides early diagnosis. b. Maternal serum alpha-fetoprotein (MSAFP) screening is recommended only for women at risk for neural tube defects. c. Percutaneous umbilical blood sampling (PUBS) is one of the triple-marker tests for Down syndrome. d. MSAFP is a screening tool only; it identifies candidates for more definitive procedures.

d. MSAFP is a screening tool only; it identifies candidates for more definitive procedures.

Maternal serum alpha-fetoprotein (MSAFP) screening indicates an elevated level. MSAFP screening is repeated and again is reported as higher than normal. What would be the next step in the assessment sequence to determine the well-being of the fetus? a. Percutaneous umbilical blood sampling (PUBS) b. Ultrasound for fetal anomalies c. Biophysical profile (BPP) for fetal well-being d. Amniocentesis for genetic anomalies

b. Ultrasound for fetal anomalies

While working with the pregnant woman in her first trimester, the nurse is aware that chorionic villus sampling (CVS) can be performed during pregnancy at: a. 4 weeks b. 8 weeks c. 10 weeks d. 14 weeks

c. 10 weeks

At 35 weeks of pregnancy a woman experiences preterm labor. Tocolytics are administered and she is placed on bed rest, but she continues to experience regular uterine contractions, and her cervix is beginning to dilate and efface. What would be an important test for fetal well-being at this time? a. Percutaneous umbilical blood sampling (PUBS) b. Ultrasound for fetal size c. Amniocentesis for fetal lung maturity d. Nonstress test (NST)

c. Amniocentesis for fetal lung maturity

In the past, factors to determine whether a woman was likely to develop a high risk pregnancy were evaluated primarily from a medical point of view. A broader, more comprehensive approach to high-risk pregnancy has been adopted today. There are now four categories based on threats to the health of the woman and the outcome of pregnancy. Which of the following is not one of these categories? a. Biophysical b. Psychosocial c. Geographic d. Environmental

c. Geographic

The nurse providing care for the antepartum woman should understand that contraction stress test (CST): a. Sometimes uses vibroacoustic stimulation. b. Is an invasive test; however, contractions are stimulated. c. Is considered negative if no late decelerations are observed with the contractions. d. Is more effective than nonstress test (NST) if the membranes have already been ruptured.

c. Is considered negative if no late decelerations are observed with the contractions.

Premature rupture of membranes a. Polyhydramnios b. Intrauterine growth restriction (maternal cause) c. Oligohydramnios d. Chromosomal abnormalities e. Intrauterine growth restriction (fetoplacental cause)

c. Oligohydramnios

When nurses help their expectant mothers assess the daily fetal movement counts, they should be aware that: a. Alcohol or cigarette smoke can irritate the fetus into greater activity. b. Kick counts should be taken every half hour and averaged every 6 hours, with every other 6-hour stretch off. c. The fetal alarm signal should go off when fetal movements stop entirely for 12 hours. d. Obese mothers familiar with their bodies can assess fetal movement as well as average-size women.

c. The fetal alarm signal should go off when fetal movements stop entirely for 12 hours.

With regard to amniocentesis, nurses should be aware that: a. Because of new imaging techniques, amniocentesis is now possible in the first trimester. b. Despite the use of ultrasound, complications still occur in the mother or infant in 5% to 10% of cases. c. The shake test, or bubble stability test, is a quick means of determining fetal maturity. d. The presence of meconium in the amniotic fluid is always cause for concern.

c. The shake test, or bubble stability test, is a quick means of determining fetal maturity.

Advanced maternal age a. Polyhydramnios b. Intrauterine growth restriction (maternal cause) c. Oligohydramnios d. Chromosomal abnormalities e. Intrauterine growth restriction (fetoplacental cause)

d. Chromosomal abnormalities

A woman arrives at the clinic seeking confirmation that she is pregnant. The following information is obtained: She is 24 years old with a body mass index (BMI) of 17.5. She admits to having used cocaine several times during the past year and drinks alcohol occasionally. Her blood pressure (BP) is 108/70 mm Hg, her pulse rate is 72 beats/min, and her respiratory rate is 16 breaths/min. The family history is positive for diabetes mellitus and cancer. Her sister recently gave birth to an infant with a neural tube defect (NTD). Which characteristics place the woman in a high risk category? a. Blood pressure, age, BMI b. Drug/alcohol use, age, family history c. Family history, blood pressure, BMI d. Family history, BMI, drug/alcohol abuse

d. Family history, BMI, drug/alcohol abuse

The nurse recognizes that a nonstress test (NST) in which two or more fetal heart rate (FHR) accelerations of 15 beats/min or more occur with fetal movement in a 20-minute period is: a. Nonreactive b. Positive c. Negative d. Reactive

d. Reactive

In comparing the abdominal and transvaginal methods of ultrasound examination, nurses should explain to their clients that: a. Both require the woman to have a full bladder. b. The abdominal examination is more useful in the first trimester. c. Initially the transvaginal examination can be painful. d. The transvaginal examination allows pelvic anatomy to be evaluated in greater detail.

d. The transvaginal examination allows pelvic anatomy to be evaluated in greater detail.

A 40-year-old woman is 10 weeks pregnant. Which diagnostic tool would be appropriate to suggest to her at this time? a. Biophysical profile (BPP) b. Amniocentesis c. Maternal serum alpha-fetoprotein (MSAFP) screening d. Transvaginal ultrasound

d. Transvaginal ultrasound

Abnormal placenta development a. Polyhydramnios b. Intrauterine growth restriction (maternal cause) c. Oligohydramnios d. Chromosomal abnormalities e. Intrauterine growth restriction (fetoplacental cause)

e. Intrauterine growth restriction (fetoplacental cause)


Set pelajaran terkait

Foreign Policy in countries (rest of unit 1 lectures)

View Set

IGCSE Physics: Electronics components

View Set

PrepU- Chapter 3: Collecting Objective Data: The Physical Examination

View Set

Edexcel AS/A Level Business - Theme 2.1.1 Internal Finance

View Set